Programs & Examples On #Functional java

Functional Java is is an open source library to learn and implement functional programming concepts in Java.

How Do I Take a Screen Shot of a UIView?

Swift 4 updated :

extension UIView {
   var screenShot: UIImage?  {
        if #available(iOS 10, *) {
            let renderer = UIGraphicsImageRenderer(bounds: self.bounds)
            return renderer.image { (context) in
                self.layer.render(in: context.cgContext)
            }
        } else {
            UIGraphicsBeginImageContextWithOptions(bounds.size, false, 5);
            if let _ = UIGraphicsGetCurrentContext() {
                drawHierarchy(in: bounds, afterScreenUpdates: true)
                let screenshot = UIGraphicsGetImageFromCurrentImageContext()
                UIGraphicsEndImageContext()
                return screenshot
            }
            return nil
        }
    }
}

Using LIKE in an Oracle IN clause

Yes, you can use this query (Instead of 'Specialist' and 'Developer', type any strings you want separated by comma and change employees table with your table)

SELECT * FROM employees em
    WHERE EXISTS (select 1 from table(sys.dbms_debug_vc2coll('Specialist', 'Developer')) mt where em.job like ('%' || mt.column_value || '%'));

Why my query is better than the accepted answer: You don't need a CREATE TABLE permission to run it. This can be executed with just SELECT permissions.

Node.js spawn child process and get terminal output live

I found myself requiring this functionality often enough that I packaged it into a library called std-pour. It should let you execute a command and view the output in real time. To install simply:

npm install std-pour

Then it's simple enough to execute a command and see the output in realtime:

const { pour } = require('std-pour');
pour('ping', ['8.8.8.8', '-c', '4']).then(code => console.log(`Error Code: ${code}`));

It's promised based so you can chain multiple commands. It's even function signature-compatible with child_process.spawn so it should be a drop in replacement anywhere you're using it.

Check if page gets reloaded or refreshed in JavaScript

I have wrote this function to check both methods using old window.performance.navigation and new performance.getEntriesByType("navigation") in same time:

function navigationType(){

    var result;
    var p;

    if (window.performance.navigation) {
        result=window.performance.navigation;
        if (result==255){result=4} // 4 is my invention!
    }

    if (window.performance.getEntriesByType("navigation")){
       p=window.performance.getEntriesByType("navigation")[0].type;

       if (p=='navigate'){result=0}
       if (p=='reload'){result=1}
       if (p=='back_forward'){result=2}
       if (p=='prerender'){result=3} //3 is my invention!
    }
    return result;
}

Result description:

0: clicking a link, Entering the URL in the browser's address bar, form submission, Clicking bookmark, initializing through a script operation.

1: Clicking the Reload button or using Location.reload()

2: Working with browswer history (Bakc and Forward).

3: prerendering activity like <link rel="prerender" href="//example.com/next-page.html">

4: any other method.

Ternary operator ?: vs if...else

Just to be a bit left handed...

x ? y : x = value

will assign value to y if x is not 0 (false).

Reading InputStream as UTF-8

I ran into the same problem every time it finds a special character marks it as ??. to solve this, I tried using the encoding: ISO-8859-1

BufferedReader br = new BufferedReader(new InputStreamReader(new FileInputStream("txtPath"),"ISO-8859-1"));

while ((line = br.readLine()) != null) {

}

I hope this can help anyone who sees this post.

notifyDataSetChanged not working on RecyclerView

Just to complement the other answers as I don't think anyone mentioned this here: notifyDataSetChanged() should be executed on the main thread (other notify<Something> methods of RecyclerView.Adapter as well, of course)

From what I gather, since you have the parsing procedures and the call to notifyDataSetChanged() in the same block, either you're calling it from a worker thread, or you're doing JSON parsing on main thread (which is also a no-no as I'm sure you know). So the proper way would be:

protected void parseResponse(JSONArray response, String url) {
    // insert dummy data for demo
    // <yadda yadda yadda>
    mBusinessAdapter = new BusinessAdapter(mBusinesses);
    // or just use recyclerView.post() or [Fragment]getView().post()
    // instead, but make sure views haven't been destroyed while you were
    // parsing
    new Handler(Looper.getMainLooper()).post(new Runnable() {
        public void run() {
            mBusinessAdapter.notifyDataSetChanged();
        }
    });

}

PS Weird thing is, I don't think you get any indications about the main thread thing from either IDE or run-time logs. This is just from my personal observations: if I do call notifyDataSetChanged() from a worker thread, I don't get the obligatory Only the original thread that created a view hierarchy can touch its views message or anything like that - it just fails silently (and in my case one off-main-thread call can even prevent succeeding main-thread calls from functioning properly, probably because of some kind of race condition)

Moreover, neither the RecyclerView.Adapter api reference nor the relevant official dev guide explicitly mention the main thread requirement at the moment (the moment is 2017) and none of the Android Studio lint inspection rules seem to concern this issue either.

But, here is an explanation of this by the author himself

Proper way to use **kwargs in Python

If you want to combine this with *args you have to keep *args and **kwargs at the end of the definition.

So:

def method(foo, bar=None, *args, **kwargs):
    do_something_with(foo, bar)
    some_other_function(*args, **kwargs)

Find Oracle JDBC driver in Maven repository

This worked for me like charm. I went through multiple ways but then this helped me. Make sure you follow each step and name the XML files exactly same.

https://blogs.oracle.com/dev2dev/get-oracle-jdbc-drivers-and-ucp-from-oracle-maven-repository-without-ides

The process is a little tedious but yes it does work.

Parse Error: Adjacent JSX elements must be wrapped in an enclosing tag

I think the complication may also occur when trying to nest multiple Divs within the return statement. You may wish to do this to ensure your components render as block elements.

Here's an example of correctly rendering a couple of components, using multiple divs.

return (
  <div>
    <h1>Data Information</H1>
    <div>
      <Button type="primary">Create Data</Button>
    </div>
  </div>
)

Get counts of all tables in a schema

This can be done with a single statement and some XML magic:

select table_name, 
       to_number(extractvalue(xmltype(dbms_xmlgen.getxml('select count(*) c from '||owner||'.'||table_name)),'/ROWSET/ROW/C')) as count
from all_tables
where owner = 'FOOBAR'

Flutter: Setting the height of the AppBar

Cinn's answer is great, but there's one thing wrong with it.

The PreferredSize widget will start immediately at the top of the screen, without accounting for the status bar, so some of its height will be shadowed by the status bar's height. This also accounts for the side notches.

The solution: Wrap the preferredSize's child with a SafeArea

appBar: PreferredSize(
  //Here is the preferred height.
  preferredSize: Size.fromHeight(50.0),
  child: SafeArea(
    child: AppBar(
      flexibleSpace: ...
    ),
  ),
),

If you don't wanna use the flexibleSpace property, then there's no need for all that, because the other properties of the AppBar will account for the status bar automatically.

Getting all names in an enum as a String[]

I would write it like this

public static String[] names() {

    java.util.LinkedList<String> list = new LinkedList<String>();
    for (State s : State.values()) {
        list.add(s.name());
    }

    return list.toArray(new String[list.size()]);
}

How to generate and manually insert a uniqueidentifier in sql server?

ApplicationId must be of type UniqueIdentifier. Your code works fine if you do:

DECLARE @TTEST TABLE
(
  TEST UNIQUEIDENTIFIER
)

DECLARE @UNIQUEX UNIQUEIDENTIFIER
SET @UNIQUEX = NEWID();

INSERT INTO @TTEST
(TEST)
VALUES
(@UNIQUEX);

SELECT * FROM @TTEST

Therefore I would say it is safe to assume that ApplicationId is not the correct data type.

What is the default maximum heap size for Sun's JVM from Java SE 6?

java 1.6.0_21 or later, or so...

$ java -XX:+PrintFlagsFinal -version 2>&1 | grep MaxHeapSize
uintx MaxHeapSize                         := 12660904960      {product}

It looks like the min(1G) has been removed.

Or on Windows using findstr

C:\>java -XX:+PrintFlagsFinal -version 2>&1 | findstr MaxHeapSize

PHP Get all subdirectories of a given directory

Here is how you can retrieve only directories with GLOB:

$directories = glob($somePath . '/*' , GLOB_ONLYDIR);

Angular 2 'component' is not a known element

The problem in my case was missing component declaration in the module, but even after adding the declaration the error persisted. I had stop the server and rebuild the entire project in VS Code for the error to go away.

How to programmatically tell if a Bluetooth device is connected?

BluetoothAdapter.getDefaultAdapter().isEnabled -> returns true when bluetooth is open

val audioManager = this.getSystemService(Context.AUDIO_SERVICE) as AudioManager

audioManager.isBluetoothScoOn -> returns true when device connected

Laravel 4: how to "order by" using Eloquent ORM

This is how I would go about it.

$posts = $this->post->orderBy('id', 'DESC')->get();

Django Rest Framework -- no module named rest_framework

In my case, I had installed it in the virtualenv but forgot to activate the virtualenv while running the command

 python3 manage.py makemigrations 

So in my case I had to just activate the environment and then run the command

source [virtualenv folder-name]/bin/activate
python3 manage.py makemigrations

This solved my problem.

Error sending json in POST to web API service

another tip...where to add "content-type: application/json"...to the textbox field on the Composer/Parsed tab. There are 3 lines already filled in there, so I added this Content-type as the 4th line. That made the Post work.

Modifying local variable from inside lambda

Yes, you can modify local variables from inside lambdas (in the way shown by the other answers), but you should not do it. Lambdas have been made for functional style of programming and this means: No side effects. What you want to do is considered bad style. It is also dangerous in case of parallel streams.

You should either find a solution without side effects or use a traditional for loop.

Is there a way to remove the separator line from a UITableView?

In interface Builder set table view separator "None"

enter image description here and those separator lines which are shown after the last cell can be remove by following approach. Best approach is to assign Empty View to tableView FooterView in viewDidLoad

self.tableView.tableFooterView = UIView()

Java - How Can I Write My ArrayList to a file, and Read (load) that file to the original ArrayList?

To save and load an arraylist of public static ArrayList data = new ArrayList ();

I used (to write)...

static void saveDatabase() {
try {

        FileOutputStream fos = new FileOutputStream("mydb.fil");
        ObjectOutputStream oos = new ObjectOutputStream(fos);
        oos.writeObject(data);
        oos.close();
        databaseIsSaved = true;         

    }
catch (IOException e) {
        e.printStackTrace();
}

} // End of saveDatabase

And used (to read) ...

static void loadDatabase() {

try {           
        FileInputStream fis = new FileInputStream("mydb.fil");
        ObjectInputStream ois = new ObjectInputStream(fis);         
        data = (ArrayList<User>)ois.readObject();
        ois.close();            
    }       
catch (IOException e) {
        System.out.println("***catch ERROR***");
        e.printStackTrace();

    }       
catch (ClassNotFoundException e) {
        System.out.println("***catch ERROR***");
        e.printStackTrace();
    }   
} // End of loadDatabase 

'nuget' is not recognized but other nuget commands working

Nuget.exe is placed at .nuget folder of your project. It can't be executed directly in Package Manager Console, but is executed by Powershell commands because these commands build custom path for themselves.

My steps to solve are:


Update

NuGet can be easily installed in your project using the following command:

Install-Package NuGet.CommandLine

How to cancel a Task in await?

Read up on Cancellation (which was introduced in .NET 4.0 and is largely unchanged since then) and the Task-Based Asynchronous Pattern, which provides guidelines on how to use CancellationToken with async methods.

To summarize, you pass a CancellationToken into each method that supports cancellation, and that method must check it periodically.

private async Task TryTask()
{
  CancellationTokenSource source = new CancellationTokenSource();
  source.CancelAfter(TimeSpan.FromSeconds(1));
  Task<int> task = Task.Run(() => slowFunc(1, 2, source.Token), source.Token);

  // (A canceled task will raise an exception when awaited).
  await task;
}

private int slowFunc(int a, int b, CancellationToken cancellationToken)
{
  string someString = string.Empty;
  for (int i = 0; i < 200000; i++)
  {
    someString += "a";
    if (i % 1000 == 0)
      cancellationToken.ThrowIfCancellationRequested();
  }

  return a + b;
}

How to check if a variable is both null and /or undefined in JavaScript

A variable cannot be both null and undefined at the same time. However, the direct answer to your question is:

if (variable != null)

One =, not two.

There are two special clauses in the "abstract equality comparison algorithm" in the JavaScript spec devoted to the case of one operand being null and the other being undefined, and the result is true for == and false for !=. Thus if the value of the variable is undefined, it's not != null, and if it's not null, it's obviously not != null.

Now, the case of an identifier not being defined at all, either as a var or let, as a function parameter, or as a property of the global context is different. A reference to such an identifier is treated as an error at runtime. You could attempt a reference and catch the error:

var isDefined = false;
try {
  (variable);
  isDefined = true;
}
catch (x) {}

I would personally consider that a questionable practice however. For global symbols that may or may be there based on the presence or absence of some other library, or some similar situation, you can test for a window property (in browser JavaScript):

var isJqueryAvailable = window.jQuery != null;

or

var isJqueryAvailable = "jQuery" in window;

Android Studio Gradle project "Unable to start the daemon process /initialization of VM"

File->Settings->compiler->check - Compile independent modules in parallel(may require larger heap size)

gradle.properties

org.gradle.jvmargs=-Xmx768m

add this end of the file.

it works for me.

if Not, uncheck - Compile independent modules in parallel, and set org.gradle.jvmargs=Xmx768m like above

Avoid web.config inheritance in child web application using inheritInChildApplications

We were getting an error related to this after a recent release of code to one of our development environments. We have an application that is a child of another application. This relationship has been working fine for YEARS until yesterday.

The problem:
We were getting a yellow stack trace error due to duplicate keys being entered. This is because both the web.config for the child and parent applications had this key. But this existed for many years like this without change. Why all of sudden its an issue now?

The solution:
The reason this was never a problem is because the keys AND values were always the same. Yesterday we updated our SQL connection strings to include the Application Name in the connection string. This made the string unique and all of sudden started to fail.

Without doing any research on the exact reason for this, I have to assume that when the child application inherits the parents web.config values, it ignores identical key/value pairs.

We were able to solve it by wrapping the connection string like this

    <location path="." inheritInChildApplications="false">
        <connectionStrings>
            <!-- Updated connection strings go here -->
        </connectionStrings>
    </location>

Edit: I forgot to mention that I added this in the PARENTS web.config. I didn't have to modify the child's web.config.

Thanks for everyones help on this, saved our butts.

Can I automatically increment the file build version when using Visual Studio?

Setting a * in the version number in AssemblyInfo or under project properties as described in the other posts does not work with all versions of Visual Studio / .NET.

Afaik it did not work in VS 2005 (but in VS 2003 and VS 2008). For VS 2005 you could use the following: Auto Increment Visual Studio 2005 version build and revision number on compile time.

But be aware that changing the version number automatically is not recommended for strong-named assemblies. The reason is that all references to such an assembly must be updated each time the referenced assembly is rebuilt due to the fact that strong-named assembly references are always a reference to a specific assembly version. Microsoft themselves change the version number of the .NET Framework assemblies only if there are changes in interfaces. (NB: I'm still searching for the link in MSDN where I read that.)

How can I get a Unicode character's code?

dear friend, Jon Skeet said you can find character Decimal codebut it is not character Hex code as it should mention in unicode, so you should represent character codes via HexCode not in Deciaml.

there is an open source tool at http://unicode.codeplex.com that provides complete information about a characer or a sentece.

so it is better to create a parser that give a char as a parameter and return ahexCode as string

public static String GetHexCode(char character)
    {
        return String.format("{0:X4}", GetDecimal(character));
    }//end

hope it help

Fit Image into PictureBox

The PictureBox.SizeMode options are missing a "fill" or "cover" mode which would be like zoom except with cropping to ensure you're filling the picture box. In CSS it's the "cover" option.

This code should enable that:

static public void fillPictureBox(PictureBox pbox, Bitmap bmp)
{
    pbox.SizeMode = PictureBoxSizeMode.Normal;
    bool source_is_wider = (float)bmp.Width / bmp.Height > (float)pbox.Width / pbox.Height;

    var resized = new Bitmap(pbox.Width, pbox.Height);
    var g = Graphics.FromImage(resized);        
    var dest_rect = new Rectangle(0, 0, pbox.Width, pbox.Height);
    Rectangle src_rect;

    if (source_is_wider)
    {
        float size_ratio = (float)pbox.Height / bmp.Height;
        int sample_width = (int)(pbox.Width / size_ratio);
        src_rect = new Rectangle((bmp.Width - sample_width) / 2, 0, sample_width, bmp.Height);
    }
    else
    {
        float size_ratio = (float)pbox.Width / bmp.Width;
        int sample_height = (int)(pbox.Height / size_ratio);
        src_rect = new Rectangle(0, (bmp.Height - sample_height) / 2, bmp.Width, sample_height);
    }

    g.DrawImage(bmp, dest_rect, src_rect, GraphicsUnit.Pixel);
    g.Dispose();

    pbox.Image = resized;
}

How to solve PHP error 'Notice: Array to string conversion in...'

What the PHP Notice means and how to reproduce it:

If you send a PHP array into a function that expects a string like: echo or print, then the PHP interpreter will convert your array to the literal string Array, throw this Notice and keep going. For example:

php> print(array(1,2,3))

PHP Notice:  Array to string conversion in 
/usr/local/lib/python2.7/dist-packages/phpsh/phpsh.php(591) :
eval()'d code on line 1
Array

In this case, the function print dumps the literal string: Array to stdout and then logs the Notice to stderr and keeps going.

Another example in a PHP script:

<?php
    $stuff = array(1,2,3);
    print $stuff;  //PHP Notice:  Array to string conversion in yourfile on line 3
?>

Correction 1: use foreach loop to access array elements

http://php.net/foreach

$stuff = array(1,2,3);
foreach ($stuff as $value) {
    echo $value, "\n";
}

Prints:

1
2
3

Or along with array keys

$stuff = array('name' => 'Joe', 'email' => '[email protected]');
foreach ($stuff as $key => $value) {
    echo "$key: $value\n";
}

Prints:

name: Joe
email: [email protected]

Note that array elements could be arrays as well. In this case either use foreach again or access this inner array elements using array syntax, e.g. $row['name']

Correction 2: Joining all the cells in the array together:

In case it's just a plain 1-demensional array, you can simply join all the cells into a string using a delimiter:

<?php
    $stuff = array(1,2,3);
    print implode(", ", $stuff);    //prints 1, 2, 3
    print join(',', $stuff);        //prints 1,2,3

Correction 3: Stringify an array with complex structure:

In case your array has a complex structure but you need to convert it to a string anyway, then use http://php.net/json_encode

$stuff = array('name' => 'Joe', 'email' => '[email protected]');
print json_encode($stuff);

Prints

{"name":"Joe","email":"[email protected]"}

A quick peek into array structure: use the builtin php functions

If you want just to inspect the array contents for the debugging purpose, use one of the following functions. Keep in mind that var_dump is most informative of them and thus usually being preferred for the purpose

examples

$stuff = array(1,2,3);
print_r($stuff);
$stuff = array(3,4,5);
var_dump($stuff);

Prints:

Array
(
    [0] => 1
    [1] => 2
    [2] => 3
)
array(3) {
  [0]=>
  int(3)
  [1]=>
  int(4)
  [2]=>
  int(5)
}

No internet on Android emulator - why and how to fix?

Check your internet settings, firewalls and such may be blocking it, I know when I was working on it in college they were blocking the port number but I've never had any trouble on my home machines

How do I accomplish an if/else in mustache.js?

Note, you can use {{.}} to render the current context item.

{{#avatar}}{{.}}{{/avatar}}

{{^avatar}}missing{{/avatar}}

Dynamically allocating an array of objects

For building containers you obviously want to use one of the standard containers (such as a std::vector). But this is a perfect example of the things you need to consider when your object contains RAW pointers.

If your object has a RAW pointer then you need to remember the rule of 3 (now the rule of 5 in C++11).

  • Constructor
  • Destructor
  • Copy Constructor
  • Assignment Operator
  • Move Constructor (C++11)
  • Move Assignment (C++11)

This is because if not defined the compiler will generate its own version of these methods (see below). The compiler generated versions are not always useful when dealing with RAW pointers.

The copy constructor is the hard one to get correct (it's non trivial if you want to provide the strong exception guarantee). The Assignment operator can be defined in terms of the Copy Constructor as you can use the copy and swap idiom internally.

See below for full details on the absolute minimum for a class containing a pointer to an array of integers.

Knowing that it is non trivial to get it correct you should consider using std::vector rather than a pointer to an array of integers. The vector is easy to use (and expand) and covers all the problems associated with exceptions. Compare the following class with the definition of A below.

class A
{ 
    std::vector<int>   mArray;
    public:
        A(){}
        A(size_t s) :mArray(s)  {}
};

Looking at your problem:

A* arrayOfAs = new A[5];
for (int i = 0; i < 5; ++i)
{
    // As you surmised the problem is on this line.
    arrayOfAs[i] = A(3);

    // What is happening:
    // 1) A(3) Build your A object (fine)
    // 2) A::operator=(A const&) is called to assign the value
    //    onto the result of the array access. Because you did
    //    not define this operator the compiler generated one is
    //    used.
}

The compiler generated assignment operator is fine for nearly all situations, but when RAW pointers are in play you need to pay attention. In your case it is causing a problem because of the shallow copy problem. You have ended up with two objects that contain pointers to the same piece of memory. When the A(3) goes out of scope at the end of the loop it calls delete [] on its pointer. Thus the other object (in the array) now contains a pointer to memory that has been returned to the system.

The compiler generated copy constructor; copies each member variable by using that members copy constructor. For pointers this just means the pointer value is copied from the source object to the destination object (hence shallow copy).

The compiler generated assignment operator; copies each member variable by using that members assignment operator. For pointers this just means the pointer value is copied from the source object to the destination object (hence shallow copy).

So the minimum for a class that contains a pointer:

class A
{
    size_t     mSize;
    int*       mArray;
    public:
         // Simple constructor/destructor are obvious.
         A(size_t s = 0) {mSize=s;mArray = new int[mSize];}
        ~A()             {delete [] mArray;}

         // Copy constructor needs more work
         A(A const& copy)
         {
             mSize  = copy.mSize;
             mArray = new int[copy.mSize];

             // Don't need to worry about copying integers.
             // But if the object has a copy constructor then
             // it would also need to worry about throws from the copy constructor.
             std::copy(&copy.mArray[0],&copy.mArray[c.mSize],mArray);

         }

         // Define assignment operator in terms of the copy constructor
         // Modified: There is a slight twist to the copy swap idiom, that you can
         //           Remove the manual copy made by passing the rhs by value thus
         //           providing an implicit copy generated by the compiler.
         A& operator=(A rhs) // Pass by value (thus generating a copy)
         {
             rhs.swap(*this); // Now swap data with the copy.
                              // The rhs parameter will delete the array when it
                              // goes out of scope at the end of the function
             return *this;
         }
         void swap(A& s) noexcept
         {
             using std::swap;
             swap(this.mArray,s.mArray);
             swap(this.mSize ,s.mSize);
         }

         // C++11
         A(A&& src) noexcept
             : mSize(0)
             , mArray(NULL)
         {
             src.swap(*this);
         }
         A& operator=(A&& src) noexcept
         {
             src.swap(*this);     // You are moving the state of the src object
                                  // into this one. The state of the src object
                                  // after the move must be valid but indeterminate.
                                  //
                                  // The easiest way to do this is to swap the states
                                  // of the two objects.
                                  //
                                  // Note: Doing any operation on src after a move 
                                  // is risky (apart from destroy) until you put it 
                                  // into a specific state. Your object should have
                                  // appropriate methods for this.
                                  // 
                                  // Example: Assignment (operator = should work).
                                  //          std::vector() has clear() which sets
                                  //          a specific state without needing to
                                  //          know the current state.
             return *this;
         }   
 }

How do I calculate the MD5 checksum of a file in Python?

You can calculate the checksum of a file by reading the binary data and using hashlib.md5().hexdigest(). A function to do this would look like the following:

def File_Checksum_Dis(dirname):
    
    if not os.path.exists(dirname):
        print(dirname+" directory is not existing");
    
    for fname in os.listdir(dirname):
        if not fname.endswith('~'):
            fnaav = os.path.join(dirname, fname);
            fd = open(fnaav, 'rb');
            data = fd.read();
            fd.close();
        
            print("-"*70);
            print("File Name is: ",fname);          
            print(hashlib.md5(data).hexdigest())
            print("-"*70);
                

Convert a positive number to negative in C#

The same way you make anything else negative: put a negative sign in front of it.

var positive = 6;
var negative = -positive;

How to sort a list/tuple of lists/tuples by the element at a given index?

In order to sort a list of tuples (<word>, <count>), for count in descending order and word in alphabetical order:

data = [
('betty', 1),
('bought', 1),
('a', 1),
('bit', 1),
('of', 1),
('butter', 2),
('but', 1),
('the', 1),
('was', 1),
('bitter', 1)]

I use this method:

sorted(data, key=lambda tup:(-tup[1], tup[0]))

and it gives me the result:

[('butter', 2),
('a', 1),
('betty', 1),
('bit', 1),
('bitter', 1),
('bought', 1),
('but', 1),
('of', 1),
('the', 1),
('was', 1)]

How to enable production mode?

To enable production mode in angular 6.X.X Just go to environment file

Like this path

Your path: project>\src\environments\environment.ts

Change production: false from :

export const environment = {
  production: false
};

To

export const environment = {
  production: true
};

enter image description here

Invoking a static method using reflection

String methodName= "...";
String[] args = {};

Method[] methods = clazz.getMethods();
for (Method m : methods) {
    if (methodName.equals(m.getName())) {
        // for static methods we can use null as instance of class
        m.invoke(null, new Object[] {args});
        break;
    }
}

Unable to locate tools.jar

it has been solved with me in windows os by setting the JAVA_HOME variable before running as follows:

set JAVA_HOME=C:\Program Files\Java\jdk1.8.0_111

Using Sockets to send and receive data

the easiest way to do this is to wrap your sockets in ObjectInput/OutputStreams and send serialized java objects. you can create classes which contain the relevant data, and then you don't need to worry about the nitty gritty details of handling binary protocols. just make sure that you flush your object streams after you write each object "message".

Assigning variables with dynamic names in Java

You should use List or array instead

List<Integer> list = new ArrayList<Integer>();
list.add(1);
list.add(2);
list.add(3);

Or

int[] arr  = new int[10];
arr[0]=1;
arr[1]=2;

Or even better

Map<String, Integer> map = new HashMap<String, Integer>();
map.put("n1", 1);
map.put("n2", 2);

//conditionally get 
map.get("n1");

How to send a HTTP OPTIONS request from the command line?

The curl installed by default in Debian supports HTTPS since a great while back. (a long time ago there were two separate packages, one with and one without SSL but that's not the case anymore)

OPTIONS /path

You can send an OPTIONS request with curl like this:

curl -i -X OPTIONS http://example.org/path

You may also use -v instead of -i to see more output.

OPTIONS *

To send a plain * (instead of the path, see RFC 7231) with the OPTIONS method, you need curl 7.55.0 or later as then you can run a command line like:

curl -i --request-target "*" -X OPTIONS http://example.org

IntelliJ: Working on multiple projects

Press "F4" on windows which will open up "Project Structure" and then click "+" icon or "Alt + Insert" to select a new project to be imported; then click OK button...

Create a shortcut on Desktop

For Windows Vista/7/8/10, you can create a symlink instead via mklink.

Process.Start("cmd.exe", $"/c mklink {linkName} {applicationPath}");

Alternatively, call CreateSymbolicLink via P/Invoke.

Undefined reference to `pow' and `floor'

To find the point where to add the -lm in Eclipse-IDE is really horrible, so it took me some time.

If someone else also uses Edlipse, here's the way how to add the command:

Project -> Properties -> C/C++ Build -> Settings -> GCC C Linker -> Miscelleaneous -> Linker flags: in this field add the command -lm

How do I obtain crash-data from my Android application?

There is this android library called Sherlock. It gives you the full report of crash along with device and application information. Whenever a crash occurs, it displays a notification in the notification bar and on clicking of the notification, it opens the crash details. You can also share crash details with others via email or other sharing options.

Installation

android {
    dataBinding {
      enabled = true
    }
}

compile('com.github.ajitsing:sherlock:1.0.0@aar') {
    transitive = true
}

Demo

enter image description here

Full Page <iframe>

Put this in your CSS.

iframe {
  width: 100%;
  height: 100vh;
}

Query a parameter (postgresql.conf setting) like "max_connections"

You can use SHOW:

SHOW max_connections;

This returns the currently effective setting. Be aware that it can differ from the setting in postgresql.conf as there are a multiple ways to set run-time parameters in PostgreSQL. To reset the "original" setting from postgresql.conf in your current session:

RESET max_connections;

However, not applicable to this particular setting. The manual:

This parameter can only be set at server start.

To see all settings:

SHOW ALL;

There is also pg_settings:

The view pg_settings provides access to run-time parameters of the server. It is essentially an alternative interface to the SHOW and SET commands. It also provides access to some facts about each parameter that are not directly available from SHOW, such as minimum and maximum values.

For your original request:

SELECT *
FROM   pg_settings
WHERE  name = 'max_connections';

Finally, there is current_setting(), which can be nested in DML statements:

SELECT current_setting('max_connections');

Related:

Long press on UITableView

Looks to be more efficient to add the recognizer directly to the cell as shown here:

Tap&Hold for TableView Cells, Then and Now

(scroll to the example at the bottom)

Jackson - best way writes a java list to a json array

This is overly complicated, Jackson handles lists via its writer methods just as well as it handles regular objects. This should work just fine for you, assuming I have not misunderstood your question:

public void writeListToJsonArray() throws IOException {  
    final List<Event> list = new ArrayList<Event>(2);
    list.add(new Event("a1","a2"));
    list.add(new Event("b1","b2"));

    final ByteArrayOutputStream out = new ByteArrayOutputStream();
    final ObjectMapper mapper = new ObjectMapper();

    mapper.writeValue(out, list);

    final byte[] data = out.toByteArray();
    System.out.println(new String(data));
}

'namespace' but is used like a 'type'

namespace TestApplication // Remove .Controller
{
    public class HomeController : Controller
    {
       public ActionResult Index()
        {
            return View();
        }
    }
}

Remove the controller word from namepsace

How to convert R Markdown to PDF?

Follow these simple steps :

1: In the Rmarkdown script run Knit(Ctrl+Shift+K) 2: Then after the html markdown is opened click Open in Browser(top left side) and the html is opened in your web browser 3: Then use Ctrl+P and save as PDF .

Safari 3rd party cookie iframe trick no longer working?

I recently hit the same issue on Safari. The solution I figured out is based on the Local Storage HTML5 API. Using Local Storage you could emulate cookies.

Here's my blog post with details: http://log.scalemotion.com/2012/10/how-to-trick-safari-and-set-3rd-party.html

DB2 SQL error: SQLCODE: -206, SQLSTATE: 42703

That only means that an undefined column or parameter name was detected. The errror that DB2 gives should point what that may be:

DB2 SQL Error: SQLCODE=-206, SQLSTATE=42703, SQLERRMC=[THE_UNDEFINED_COLUMN_OR_PARAMETER_NAME], DRIVER=4.8.87

Double check your table definition. Maybe you just missed adding something.

I also tried google-ing this problem and saw this:

http://www.coderanch.com/t/515475/JDBC/databases/sql-insert-statement-giving-sqlcode

no suitable HttpMessageConverter found for response type

A refinement of Vadim Zin4uk's answer is just to use the existing GsonHttpMessageConverter class but invoke the setSupportedMediaTypes() setter.

For spring boot apps, this results into adding to following to your configuration classes:

@Bean
public GsonHttpMessageConverter gsonHttpMessageConverter(Gson gson) {
    GsonHttpMessageConverter converter = new GsonHttpMessageConverter();
    converter.setGson(gson);
    List<MediaType> supportedMediaTypes = converter.getSupportedMediaTypes();
    if (! supportedMediaTypes.contains(TEXT_PLAIN)) {
        supportedMediaTypes = new ArrayList<>(supportedMediaTypes);
        supportedMediaTypes.add(TEXT_PLAIN);
        converter.setSupportedMediaTypes(supportedMediaTypes);
    }
    return converter;
}

How update the _id of one MongoDB Document?

Here I have a solution that avoid multiple requests, for loops and old document removal.

You can easily create a new idea manually using something like:_id:ObjectId() But knowing Mongo will automatically assign an _id if missing, you can use aggregate to create a $project containing all the fields of your document, but omit the field _id. You can then save it with $out

So if your document is:

{
"_id":ObjectId("5b5ed345cfbce6787588e480"),
"title": "foo",
"description": "bar"
}

Then your query will be:

    db.getCollection('myCollection').aggregate([
        {$match:
             {_id: ObjectId("5b5ed345cfbce6787588e480")}
        }        
        {$project:
            {
             title: '$title',
             description: '$description'             
            }     
        },
        {$out: 'myCollection'}
    ])

How to remove folders with a certain name

I had more than 100 files like

log-12
log-123
log-34
....

above answers did not work for me

but the following command helped me.

find . -name "log-*" -exec rm  -rf {} \;

i gave -type as . so it deletes both files and folders which starts with log-

and rm -rf deletes folders recursively even it has files.

if you want folders alone

find -type d -name "log-*" -exec rm  -rf {} \;

files alone

find -type f -name "log-*" -exec rm  -rf {} \;

Undefined symbols for architecture i386: _OBJC_CLASS_$_SKPSMTPMessage", referenced from: error

Just enable the "Build Active Architecture Only" option

in PROJECT >> Build Settings >> Build Active Architecture Only >> Yes

screenshot of the path to the configuration

remember to check if they are also enabled in each target

Replace multiple strings at once

A simple forEach loop solves this quite well:

let text = 'the red apple and the green ball';
const toStrip = ['red', 'green'];
toStrip.forEach(x => {
   text = text.replace(x, '');
});

console.log(text);
// logs -> the apple and the ball

How to use Typescript with native ES6 Promises

As of TypeScript 2.0 you can include typings for native promises by including the following in your tsconfig.json

"compilerOptions": {
    "lib": ["es5", "es2015.promise"]
}

This will include the promise declarations that comes with TypeScript without having to set the target to ES6.

Manually highlight selected text in Notepad++

"Select your text, right click, then choose Style Token and then using 1st style (2nd style, etc …). At the moment is not possible to save the style tokens but there is an idea pending on Idea torrent you may vote for if your are interested in that."

It should be default, but it might be hidden.

"It might be that something happened to your contextMenu.xml so that you only get the basic standard. Have a look in NPPs config folder (%appdata%\Notepad++\) if the contextMenu.xml is there. If no: that would be the answer; if yes: it might be defect. Anyway you can grab the original standart contextMenu.xml from here and place it into the config folder (or replace the existing xml). Start NPP and you should have quite a long context menu. Tip: have a look at the contextmenu.xml itself - because you're allowed to change it to your own needs."

See this for more information

Phone number formatting an EditText in Android

Simply use the PhoneNumberFormattingTextWatcher, just call:

editText.addTextChangedListener(new PhoneNumberFormattingTextWatcher());

Addition
To be clear, PhoneNumberFormattingTextWatcher's backbone is the PhoneNumberUtils class. The difference is the TextWatcher maintains the EditText while you must call PhoneNumberUtils.formatNumber() every time you change its contents.

How to purge tomcat's cache when deploying a new .war file? Is there a config setting?

I had the same issue twice, but in the second time I realized it wasn't a problem on Tomcat at all.. Try to delete the cache of your browser, refresh the page and see if the new version of the page on your server is being shown up. It worked with me.

how to use Blob datatype in Postgres

I think this is the most comprehensive answer on the PostgreSQL wiki itself: https://wiki.postgresql.org/wiki/BinaryFilesInDB

Read the part with the title 'What is the best way to store the files in the Database?'

Extend contigency table with proportions (percentages)

Your code doesn't seem so ugly to me...
however, an alternative (not much better) could be e.g. :

df <- data.frame(table(yn))
colnames(df) <- c('Smoker','Freq')
df$Perc <- df$Freq / sum(df$Freq) * 100

------------------
  Smoker Freq Perc
1     No   19 47.5
2    Yes   21 52.5

How to set True as default value for BooleanField on Django?

In DJango 3.0 the default value of a BooleanField in model.py is set like this:

class model_name(models.Model):
example_name = models.BooleanField(default=False)

C#: How would I get the current time into a string?

I'd just like to point out something in these answers. In a date/time format string, '/' will be replaced with whatever the user's date separator is, and ':' will be replaced with whatever the user's time separator is. That is, if I've defined my date separator to be '.' (in the Regional and Language Options control panel applet, "intl.cpl"), and my time separator to be '?' (just pretend I'm crazy like that), then

DateTime.Now.ToString("MM/dd/yyyy h:mm tt")

would return

01.05.2009 6?01 PM

In most cases, this is what you want, because you want to respect the user's settings. If, however, you require the format be something specific (say, if it's going to parsed back out by somebody else down the wire), then you need to escape these special characters:

DateTime.Now.ToString("MM\\/dd\\/yyyy h\\:mm tt")

or

DateTime.Now.ToString(@"MM\/dd\/yyyy h\:mm tt")

which would now return

01/05/2009 6:01 PM

EDIT:

Then again, if you really want to respect the user's settings, you should use one of the standard date/time format strings, so that you respect not only the user's choices of separators, but also the general format of the date and/or time.

DateTime.Now.ToShortDateString()
DateTime.Now.ToString("d")

Both would return "1/5/2009" using standard US options, or "05/01/2009" using standard UK options, for instance.

DateTime.Now.ToLongDateString()
DateTime.Now.ToString("D")

Both would return "Monday, January 05, 2009" in US locale, or "05 January 2009" in UK.

DateTime.Now.ToShortTimeString()
DateTime.Now.ToString("t");

"6:01 PM" in US, "18:01" in UK.

DateTime.Now.ToLongTimeString()
DateTime.Now.ToString("T");

"6:01:04 PM" in US, "18:01:04" in UK.

DateTime.Now.ToString()
DateTime.Now.ToString("G");

"1/5/2009 6:01:04 PM" in US, "05/01/2009 18:01:04" in UK.

Many other options are available. See docs for standard date and time format strings and custom date and time format strings.

How to specify multiple conditions in an if statement in javascript

the whole if should be enclosed in brackets and the or operator is || an not !!, so

if ((Type == 2 && PageCount == 0) || (Type == 2 && PageCount == '')) { ...

Execute PHP function with onclick

First, understand that you have three languages working together:

  • PHP: It only runs by the server and responds to requests like clicking on a link (GET) or submitting a form (POST).

  • HTML & JavaScript: It only runs in someone's browser (excluding NodeJS).

I'm assuming your file looks something like:

<!DOCTYPE HTML>
<html>
<?php
  function runMyFunction() {
    echo 'I just ran a php function';
  }

  if (isset($_GET['hello'])) {
    runMyFunction();
  }
?>

Hello there!
<a href='index.php?hello=true'>Run PHP Function</a>
</html>

Because PHP only responds to requests (GET, POST, PUT, PATCH, and DELETE via $_REQUEST), this is how you have to run a PHP function even though they're in the same file. This gives you a level of security, "Should I run this script for this user or not?".

If you don't want to refresh the page, you can make a request to PHP without refreshing via a method called Asynchronous JavaScript and XML (AJAX).

That is something you can look up on YouTube though. Just search "jquery ajax"

I recommend Laravel to anyone new to start off right: http://laravel.com/

How to delete only the content of file in python

What could be easier than something like this:

import tempfile

for i in range(400):
    with tempfile.TemporaryFile() as tf:
        for j in range(1000):
            tf.write('Line {} of file {}'.format(j,i))

That creates 400 temp files and writes 1000 lines to each temp file. It executes in less than 1/2 second on my unremarkable machine. Each temp file of the total is created and deleted as the context manager opens and closes in this case. It is fast, secure, and cross platform.

Using tempfile is a lot better than trying to reinvent it.

Eclipse Indigo - Cannot install Android ADT Plugin

So I got indigo, and then : Go to Help->Install New Software Click on Add: Name: "Indigo" Location: "http://download.eclipse.org/releases/indigo" Try to install Android Development Tools (as you will see, only 1 option out of 4 will appear - this is normal for Indigo)

How to save SELECT sql query results in an array in C# Asp.net

    public void ChargingArraySelect()
    {
        int loop = 0;
        int registros = 0;

        OdbcConnection conn = WebApiConfig.conn();
        OdbcCommand query = conn.CreateCommand();

        query.CommandText = "select dataA, DataB, dataC, DataD FROM table  where dataA = 'xpto'";

        try
        {
            conn.Open();
            OdbcDataReader dr = query.ExecuteReader();

            //take the number the registers, to use into next step
            registros = dr.RecordsAffected;

            //calls an array to be populated
            Global.arrayTest = new string[registros, 4];

            while (dr.Read())
            {
                if (loop < registros)
                {
                    Global.arrayTest[i, 0] = Convert.ToString(dr["dataA"]);
                    Global.arrayTest[i, 1] = Convert.ToString(dr["dataB"]);
                    Global.arrayTest[i, 2] = Convert.ToString(dr["dataC"]);
                    Global.arrayTest[i, 3] = Convert.ToString(dr["dataD"]);
                }
                loop++;
            }
        }
    }


    //Declaration the Globais Array in Global Classs
    private static string[] uso_internoArray1;
    public static string[] arrayTest
    {
        get { return uso_internoArray1; }
        set { uso_internoArray1 = value; }
    }

Calculate MD5 checksum for a file

And if you need to calculate the MD5 to see whether it matches the MD5 of an Azure blob, then this SO question and answer might be helpful: MD5 hash of blob uploaded on Azure doesnt match with same file on local machine

Convert IEnumerable to DataTable

So, 10 years later this is still a thing :)

I've tried every answer on this page (ATOW)
and also some ILGenerator powered solutions (FastMember and Fast.Reflection).
But a compiled Lambda Expression seems to be the fastest.
At least for my use cases (on .Net Core 2.2).

This is what I am using for now:

public static class EnumerableExtensions {

    internal static Func<TClass, object> CompileGetter<TClass>(string propertyName) {
        var param = Expression.Parameter(typeof(TClass));
        var body = Expression.Convert(Expression.Property(param, propertyName), typeof(object));
        return Expression.Lambda<Func<TClass, object>>(body,param).Compile();
    }     

    public static DataTable ToDataTable<T>(this IEnumerable<T> collection) {
        var dataTable = new DataTable();
        var properties = typeof(T)
                        .GetProperties(BindingFlags.Public | BindingFlags.Instance)
                        .Where(p => p.CanRead)
                        .ToArray();

        if (properties.Length < 1) return null;
        var getters = new Func<T, object>[properties.Length];

        for (var i = 0; i < properties.Length; i++) {
            var columnType = Nullable.GetUnderlyingType(properties[i].PropertyType) ?? properties[i].PropertyType;
            dataTable.Columns.Add(properties[i].Name, columnType);
            getters[i] = CompileGetter<T>(properties[i].Name);
        }

        foreach (var row in collection) {
            var dtRow = new object[properties.Length];
            for (var i = 0; i < properties.Length; i++) {
                dtRow[i] = getters[i].Invoke(row) ?? DBNull.Value;
            }
            dataTable.Rows.Add(dtRow);
        }

        return dataTable;
    }
}

Only works with properties (not fields) but it works on Anonymous Types.

Possible to extend types in Typescript?

May be below approach will be helpful for someone TS with reactjs

interface Event {
   name: string;
   dateCreated: string;
   type: string;
}

interface UserEvent<T> extends Event<T> {
    UserId: string;
}

Calculating the sum of two variables in a batch script

According to this helpful list of operators [an operator can be thought of as a mathematical expression] found here, you can tell the batch compiler that you are manipulating variables instead of fixed numbers by using the += operator instead of the + operator.

Hope I Helped!

How do I open workbook programmatically as read-only?

Does this work?

Workbooks.Open Filename:=filepath, ReadOnly:=True

Or, as pointed out in a comment, to keep a reference to the opened workbook:

Dim book As Workbook
Set book = Workbooks.Open(Filename:=filepath, ReadOnly:=True)

How to analyse the heap dump using jmap in java

VisualVm does not come with Apple JDK. You can use VisualVM Mac Application bundle(dmg) as a separate application, to compensate for that.

SimpleXML - I/O warning : failed to load external entity

this also works:

$url = "http://www.some-url";
$ch = curl_init();
curl_setopt($ch, CURLOPT_URL, $url);
curl_setopt($ch, CURLOPT_RETURNTRANSFER, true);
$xmlresponse = curl_exec($ch);
$xml=simplexml_load_string($xmlresponse);

then I just run a forloop to grab the stuff from the nodes.

like this:`

for($i = 0; $i < 20; $i++) {
$title = $xml->channel->item[$i]->title;
$link = $xml->channel->item[$i]->link;
$desc = $xml->channel->item[$i]->description;
$html .="<div><h3>$title</h3>$link<br />$desc</div><hr>";
}
echo $html;

***note that your node names will differ, obviously..and your HTML might be structured differently...also your loop might be set to higher or lower amount of results.

How to get a web page's source code from Java

I am sure that you have found a solution somewhere over the past 2 years but the following is a solution that works for your requested site

package javasandbox;

import java.io.BufferedReader;
import java.io.IOException;
import java.io.InputStreamReader;
import java.net.HttpURLConnection;
import java.net.MalformedURLException;
import java.net.URL;

/**
*
* @author Ryan.Oglesby
*/
public class JavaSandbox {

private static String sURL;

/**
 * @param args the command line arguments
 */
public static void main(String[] args) throws MalformedURLException, IOException {
    sURL = "http://www.cumhuriyet.com.tr/?hn=298710";
    System.out.println(sURL);
    URL url = new URL(sURL);
    HttpURLConnection httpCon = (HttpURLConnection) url.openConnection();
    //set http request headers
            httpCon.addRequestProperty("Host", "www.cumhuriyet.com.tr");
            httpCon.addRequestProperty("Connection", "keep-alive");
            httpCon.addRequestProperty("Cache-Control", "max-age=0");
            httpCon.addRequestProperty("Accept", "text/html,application/xhtml+xml,application/xml;q=0.9,image/webp,*/*;q=0.8");
            httpCon.addRequestProperty("User-Agent", "Mozilla/5.0 (Windows NT 6.1; WOW64) AppleWebKit/537.36 (KHTML, like Gecko) Chrome/30.0.1599.101 Safari/537.36");
            httpCon.addRequestProperty("Accept-Encoding", "gzip,deflate,sdch");
            httpCon.addRequestProperty("Accept-Language", "en-US,en;q=0.8");
            //httpCon.addRequestProperty("Cookie", "JSESSIONID=EC0F373FCC023CD3B8B9C1E2E2F7606C; lang=tr; __utma=169322547.1217782332.1386173665.1386173665.1386173665.1; __utmb=169322547.1.10.1386173665; __utmc=169322547; __utmz=169322547.1386173665.1.1.utmcsr=stackoverflow.com|utmccn=(referral)|utmcmd=referral|utmcct=/questions/8616781/how-to-get-a-web-pages-source-code-from-java; __gads=ID=3ab4e50d8713e391:T=1386173664:S=ALNI_Mb8N_wW0xS_wRa68vhR0gTRl8MwFA; scrElm=body");
            HttpURLConnection.setFollowRedirects(false);
            httpCon.setInstanceFollowRedirects(false);
            httpCon.setDoOutput(true);
            httpCon.setUseCaches(true);

            httpCon.setRequestMethod("GET");

            BufferedReader in = new BufferedReader(new InputStreamReader(httpCon.getInputStream(), "UTF-8"));
            String inputLine;
            StringBuilder a = new StringBuilder();
            while ((inputLine = in.readLine()) != null)
                a.append(inputLine);
            in.close();

            System.out.println(a.toString());

            httpCon.disconnect();
}
}

The first day of the current month in php using date_modify as DateTime object

You can do it like this:

$firstday = date_create()->modify('first day January 2010');

How to get the squared symbol (²) to display in a string

Not sure what kind of text box you are refering to. However, I'm not sure if you can do this in a text box on a user form.

A text box on a sheet you can though.

Sheets("Sheet1").Shapes("TextBox 1").TextFrame2.TextRange.Text = "R2=" & variable
Sheets("Sheet1").Shapes("TextBox 1").TextFrame2.TextRange.Characters(2, 1).Font.Superscript = msoTrue

And same thing for an excel cell

Sheets("Sheet1").Range("A1").Characters(2, 1).Font.Superscript = True

If this isn't what you're after you will need to provide more information in your question.

EDIT: posted this after the comment sorry

Deny access to one specific folder in .htaccess

Creating index.php, index.html, index.htm is not secure. Becuse, anyone can get access on your files within specified directory by guessing files name. E.g.: http://yoursite.com/includes/file.dat So, recommended method is creating a .htaccess file to deny all visitors ;). Have fun !!

How to create an email form that can send email using html

You can't send email using javascript or html. You need server side scripts in php or other technologies to send email.

Reloading a ViewController

Reinitialise the view controller

YourViewController *vc = [[YourViewController alloc] initWithNibName:@"YourViewControllerIpad" bundle:nil];
[self.navigationController vc animated:NO];

How to select element using XPATH syntax on Selenium for Python?

Check this blog by Martin Thoma. I tested the below code on MacOS Mojave and it worked as specified.

> def get_browser():
>     """Get the browser (a "driver")."""
>     # find the path with 'which chromedriver'
>     path_to_chromedriver = ('/home/moose/GitHub/algorithms/scraping/'
>                             'venv/bin/chromedriver')
>     download_dir = "/home/moose/selenium-download/"
>     print("Is directory: {}".format(os.path.isdir(download_dir)))
> 
>     from selenium.webdriver.chrome.options import Options
>     chrome_options = Options()
>     chrome_options.add_experimental_option('prefs', {
>         "plugins.plugins_list": [{"enabled": False,
>                                   "name": "Chrome PDF Viewer"}],
>         "download": {
>             "prompt_for_download": False,
>             "default_directory": download_dir
>         }
>     })
> 
>     browser = webdriver.Chrome(path_to_chromedriver,
>                                chrome_options=chrome_options)
>     return browser

Hidden Features of Java

You can switch(this) inside method definitions of enum classes. Made me shout "whut!" loudly when I discovered that this actually works.

How to set entire application in portrait mode only?

For any Android version

From XML

You can specify android:screenOrientation="portrait" for each activity in your manifest.xml file. You cannot specify this option on the application tag.

From Java

Other option is to do it programmatically, for example in an Activity base class:

@Override
public void onCreate(Bundle savedInstanceState) {
  setRequestedOrientation(ActivityInfo.SCREEN_ORIENTATION_PORTRAIT);
}

For Android 4+ (API 14+)

Last option is to do it with activity lifecycle listeners which is only available since Android 4.0 (API 14+). Everything happens in a custom Application class:

@Override
public void onCreate() {
    super.onCreate();  
    registerActivityLifecycleCallbacks(new ActivityLifecycleAdapter() {
        @Override
        public void onActivityCreated(Activity a, Bundle savedInstanceState) {
            a.setRequestedOrientation(ActivityInfo.SCREEN_ORIENTATION_PORTRAIT);
        }
    });
}

ActivityLifecycleAdapter is just a helper class you'll need to create which will be an empty implementation of ActivityLifecycleCallbacks (so you don't have to override each and every methods of that interface when you simply need one of them).

PHP Echo a large block of text

Echoing text that contains line breaks is fine, and there's no limit on the amount of text or lines you can echo at once (save for available memory).

The error in your code is caused by the unescaped single quotes which appear in the string.

See this line:

$('input_6').hint('ex: [email protected]');

You'd need to escape those single quotes in a PHP string whether it's a single line or not.

There is another good way to echo large strings, though, and that's to close the PHP block and open it again later:

if (is_single()) {
  ?>
<link type="text/css" rel="stylesheet" href="http://jotform.com/css/styles/form.css"/><style type="text/css"> 
.form-label{
width:150px !important;
}
.form-label-left{
width:150px !important;
}
.form-line{
padding:10px;
}
.form-label-right{
width:150px !important;
}
body, html{
margin:0;
padding:0;
background:false;
}

.form-all{
margin:0px auto;
padding-top:20px;
width:650px !important;
color:Black;
font-family:Verdana;
font-size:12px;
}
</style> 

<link href="http://jotform.com/css/calendarview.css" rel="stylesheet" type="text/css" /> 
<script src="http://jotform.com/js/prototype.js" type="text/javascript"></script> 
<script src="http://jotform.com/js/protoplus.js" type="text/javascript"></script> 
<script src="http://jotform.com/js/protoplus-ui.js" type="text/javascript"></script> 
<script src="http://jotform.com/js/jotform.js?v3" type="text/javascript"></script> 
<script src="http://jotform.com/js/location.js" type="text/javascript"></script> 
<script src="http://jotform.com/js/calendarview.js" type="text/javascript"></script> 
<script type="text/javascript"> 

JotForm.init(function(){
$('input_6').hint('ex: [email protected]');
});
</script>
  <?php
}else {

}

Or another alternative, which is probably better for readability, is to put all that static HTML into another page and include() it.

Change value of input and submit form in JavaScript

No. When your input type is submit, you should have an onsubmit event declared in the markup and then do the changes you want. Meaning, have an onsubmit defined in your form tag.

Otherwise change the input type to a button and then define an onclick event for that button.

How to reference a method in javadoc?

You will find much information about JavaDoc at the Documentation Comment Specification for the Standard Doclet, including the information on the

{@link package.class#member label}

tag (that you are looking for). The corresponding example from the documentation is as follows

For example, here is a comment that refers to the getComponentAt(int, int) method:

Use the {@link #getComponentAt(int, int) getComponentAt} method.

The package.class part can be ommited if the referred method is in the current class.


Other useful links about JavaDoc are:

How to convert a string to number in TypeScript?

Easiest way is to use +strVal or Number(strVal)

Examples:

let strVal1 = "123.5"
let strVal2 = "One"
let val1a = +strVal1
let val1b = Number(strVal1)
let val1c = parseFloat(strVal1)
let val1d = parseInt(strVal1)
let val1e = +strVal1 - parseInt(strVal1)
let val2a = +strVal2

console.log("val1a->", val1a) // 123.5
console.log("val1b->", val1b) // 123.5
console.log("val1c->", val1c) // 123.5
console.log("val1d->", val1d) // 123
console.log("val1e->", val1e) // 0.5
console.log("val2a->", val2a) // NaN

npm ERR! network getaddrinfo ENOTFOUND

for some reason my error kept pointing to the "proxy" property in the config file. Which was misleading. During my troubleshooting I was trying different values for the proxy and https-proxy properties, but would only get the error stating to make sure the proxy config was set properly, and pointing to an older value.

Using, NPM CONFIG LS -L command lists all the properties and values in the config file. I was then able to see the value in question was matching the https-proxy, therefore using the https-proxy. So I changed the proxy (my company uses different ones) and then it worked. figured I would add this, as with these subtle confusing errors, every perspective on it helps.

AngularJS - Animate ng-view transitions

Check this code:

Javascript:

app.config( ["$routeProvider"], function($routeProvider){
    $routeProvider.when("/part1", {"templateUrl" : "part1"});
    $routeProvider.when("/part2", {"templateUrl" : "part2"});
    $routeProvider.otherwise({"redirectTo":"/part1"});
  }]
);

function HomeFragmentController($scope) {
    $scope.$on("$routeChangeSuccess", function (scope, next, current) {
        $scope.transitionState = "active"
    });
}

CSS:

.fragmentWrapper {
    overflow: hidden;
}

.fragment {
    position: relative;
    -moz-transition-property: left;
    -o-transition-property: left;
    -webkit-transition-property: left;
    transition-property: left;
    -moz-transition-duration: 0.1s;
    -o-transition-duration: 0.1s;
    -webkit-transition-duration: 0.1s;
    transition-duration: 0.1s
}

.fragment:not(.active) {
    left: 540px;
}

.fragment.active {
    left: 0px;
}

Main page HTML:

<div class="fragmentWrapper" data-ng-view data-ng-controller="HomeFragmentController">
</div>

Partials HTML example:

<div id="part1" class="fragment {{transitionState}}">
</div>

jQuery checkbox checked state changed event

For future reference to anyone here having difficulty, if you are adding the checkboxes dynamically, the correct accepted answer above will not work. You'll need to leverage event delegation which allows a parent node to capture bubbled events from a specific descendant and issue a callback.

// $(<parent>).on('<event>', '<child>', callback);
$(document).on('change', '.checkbox', function() {
    if(this.checked) {
      // checkbox is checked
    }
});

Note that it's almost always unnecessary to use document for the parent selector. Instead choose a more specific parent node to prevent propagating the event up too many levels.

The example below displays how the events of dynamically added dom nodes do not trigger previously defined listeners.

_x000D_
_x000D_
$postList = $('#post-list');_x000D_
_x000D_
$postList.find('h1').on('click', onH1Clicked);_x000D_
_x000D_
function onH1Clicked() {_x000D_
  alert($(this).text());_x000D_
}_x000D_
_x000D_
// simulate added content_x000D_
var title = 2;_x000D_
_x000D_
function generateRandomArticle(title) {_x000D_
  $postList.append('<article class="post"><h1>Title ' + title + '</h1></article>');_x000D_
}_x000D_
_x000D_
setTimeout(generateRandomArticle.bind(null, ++title), 1000);_x000D_
setTimeout(generateRandomArticle.bind(null, ++title), 5000);_x000D_
setTimeout(generateRandomArticle.bind(null, ++title), 10000);
_x000D_
<script src="https://ajax.googleapis.com/ajax/libs/jquery/2.1.1/jquery.min.js"></script>_x000D_
<section id="post-list" class="list post-list">_x000D_
  <article class="post">_x000D_
    <h1>Title 1</h1>_x000D_
  </article>_x000D_
  <article class="post">_x000D_
    <h1>Title 2</h1>_x000D_
  </article>_x000D_
</section>
_x000D_
_x000D_
_x000D_

While this example displays the usage of event delegation to capture events for a specific node (h1 in this case), and issue a callback for such events.

_x000D_
_x000D_
$postList = $('#post-list');_x000D_
_x000D_
$postList.on('click', 'h1', onH1Clicked);_x000D_
_x000D_
function onH1Clicked() {_x000D_
  alert($(this).text());_x000D_
}_x000D_
_x000D_
// simulate added content_x000D_
var title = 2;_x000D_
_x000D_
function generateRandomArticle(title) {_x000D_
  $postList.append('<article class="post"><h1>Title ' + title + '</h1></article>');_x000D_
}_x000D_
_x000D_
setTimeout(generateRandomArticle.bind(null, ++title), 1000); setTimeout(generateRandomArticle.bind(null, ++title), 5000); setTimeout(generateRandomArticle.bind(null, ++title), 10000);
_x000D_
<script src="https://ajax.googleapis.com/ajax/libs/jquery/2.1.1/jquery.min.js"></script>_x000D_
<section id="post-list" class="list post-list">_x000D_
  <article class="post">_x000D_
    <h1>Title 1</h1>_x000D_
  </article>_x000D_
  <article class="post">_x000D_
    <h1>Title 2</h1>_x000D_
  </article>_x000D_
</section>
_x000D_
_x000D_
_x000D_

Python: BeautifulSoup - get an attribute value based on the name attribute

6 years late to the party but I've been searching for how to extract an html element's tag attribute value, so for:

<span property="addressLocality">Ayr</span>

I want "addressLocality". I kept being directed back here, but the answers didn't really solve my problem.

How I managed to do it eventually:

>>> from bs4 import BeautifulSoup as bs

>>> soup = bs('<span property="addressLocality">Ayr</span>', 'html.parser')
>>> my_attributes = soup.find().attrs
>>> my_attributes
{u'property': u'addressLocality'}

As it's a dict, you can then also use keys and 'values'

>>> my_attributes.keys()
[u'property']
>>> my_attributes.values()
[u'addressLocality']

Hopefully it helps someone else!

How can I change the font size using seaborn FacetGrid?

For the legend, you can use this

plt.setp(g._legend.get_title(), fontsize=20)

Where g is your facetgrid object returned after you call the function making it.

How to make a class JSON serializable

jsonweb seems to be the best solution for me. See http://www.jsonweb.info/en/latest/

from jsonweb.encode import to_object, dumper

@to_object()
class DataModel(object):
  def __init__(self, id, value):
   self.id = id
   self.value = value

>>> data = DataModel(5, "foo")
>>> dumper(data)
'{"__type__": "DataModel", "id": 5, "value": "foo"}'

Date difference in years using C#

It's unclear how you want to handle fractional years, but perhaps like this:

DateTime now = DateTime.Now;
DateTime origin = new DateTime(2007, 11, 3);
int calendar_years = now.Year - origin.Year;
int whole_years = calendar_years - ((now.AddYears(-calendar_years) >= origin)? 0: 1);
int another_method = calendar_years - ((now.Month - origin.Month) * 32 >= origin.Day - now.Day)? 0: 1);

What is the maximum length of a URL in different browsers?

ASP.NET 2 and SQL Server reporting services 2005 have a limit of 2028. I found this out the hard way, where my dynamic URL generator would not pass over some parameters to a report beyond that point. This was under Internet Explorer 8.

JavaScript CSS how to add and remove multiple CSS classes to an element

Try this:

function addClass(element, value) {
  if(!element.className) {
    element.className = value;
  } else {
    newClassName = element.className;
    newClassName+= " ";
    newClassName+= value;
    element.className = newClassName;
  }
}

Similar logic could be used to make a removeClass function.

Java Embedded Databases Comparison

I realize you mentioned SQL browsing, but everything else in your question makes me want to suggest you also consider DB4O, which is a great, simple object DB.

What exactly is nullptr?

Here's the LLVM header.

// -*- C++ -*-
//===--------------------------- __nullptr --------------------------------===//
//
// Part of the LLVM Project, under the Apache License v2.0 with LLVM Exceptions.
// See https://llvm.org/LICENSE.txt for license information.
// SPDX-License-Identifier: Apache-2.0 WITH LLVM-exception
//
//===----------------------------------------------------------------------===//

#ifndef _LIBCPP_NULLPTR
#define _LIBCPP_NULLPTR

#include <__config>

#if !defined(_LIBCPP_HAS_NO_PRAGMA_SYSTEM_HEADER)
#pragma GCC system_header
#endif

#ifdef _LIBCPP_HAS_NO_NULLPTR

_LIBCPP_BEGIN_NAMESPACE_STD

struct _LIBCPP_TEMPLATE_VIS nullptr_t
{
    void* __lx;

    struct __nat {int __for_bool_;};

    _LIBCPP_INLINE_VISIBILITY _LIBCPP_CONSTEXPR nullptr_t() : __lx(0) {}
    _LIBCPP_INLINE_VISIBILITY _LIBCPP_CONSTEXPR nullptr_t(int __nat::*) : __lx(0) {}

    _LIBCPP_INLINE_VISIBILITY _LIBCPP_CONSTEXPR operator int __nat::*() const {return 0;}

    template <class _Tp>
        _LIBCPP_INLINE_VISIBILITY _LIBCPP_CONSTEXPR
        operator _Tp* () const {return 0;}

    template <class _Tp, class _Up>
        _LIBCPP_INLINE_VISIBILITY
        operator _Tp _Up::* () const {return 0;}

    friend _LIBCPP_INLINE_VISIBILITY _LIBCPP_CONSTEXPR bool operator==(nullptr_t, nullptr_t) {return true;}
    friend _LIBCPP_INLINE_VISIBILITY _LIBCPP_CONSTEXPR bool operator!=(nullptr_t, nullptr_t) {return false;}
};

inline _LIBCPP_INLINE_VISIBILITY _LIBCPP_CONSTEXPR nullptr_t __get_nullptr_t() {return nullptr_t(0);}

#define nullptr _VSTD::__get_nullptr_t()

_LIBCPP_END_NAMESPACE_STD

#else  // _LIBCPP_HAS_NO_NULLPTR

namespace std
{
    typedef decltype(nullptr) nullptr_t;
}

#endif  // _LIBCPP_HAS_NO_NULLPTR

#endif  // _LIBCPP_NULLPTR

(a great deal can be uncovered with a quick grep -r /usr/include/*`)

One thing that jumps out is the operator * overload (returning 0 is a lot friendlier than segfaulting...). Another thing is it doesn't look compatible with storing an address at all. Which, compared to how it goes slinging void*'s and passing NULL results to normal pointers as sentinel values, would obviously reduce the "never forget, it might be a bomb" factor.

How do I change the font size and color in an Excel Drop Down List?

Unfortunately, you can't change the font size or styling in a drop-down list that is created using data validation.

You can style the text in a combo box, however. Follow the instructions here: Excel Data Validation Combo Box

How to download and save an image in Android

@Droidman post is pretty comprehensive. Volley works good with small data of few kbytes. When I tried to use the 'BasicImageDownloader.java' the Android Studio gave me warning that the AsyncTask class should to be static or there could be leaks. I used Volley in another test app and that kept crashing because of leaks so I am worried about using Volley for the image downloader (images can be few 100 kB).

I used Picasso and it worked well, there is small change (probably an update on Picasso) from what is posted above. Below code worked for me:

    public static void imageDownload(Context ctx, String url){
    Picasso.get().load(yourURL)
            .into(getTarget(url));
}
private static Target getTarget(final String url){

    Target target2 = new Target() {
        @Override
        public void onBitmapLoaded(final Bitmap bitmap, Picasso.LoadedFrom from) {
            new Thread(new Runnable() {

                @Override
                public void run() {

                    File file = new File(localPath + "/"+"YourImageFile.jpg");
                    try {
                        file.createNewFile();
                        FileOutputStream ostream = new FileOutputStream(file);
                        bitmap.compress(Bitmap.CompressFormat.JPEG, 80, ostream);
                        ostream.flush();
                        ostream.close();
                    } catch (IOException e) {
                        Log.e("IOException", e.getLocalizedMessage());
                    }
                }
            }).start();
        }

        @Override
        public void onBitmapFailed(Exception e, Drawable errorDrawable) {
        }

        @Override
        public void onPrepareLoad(Drawable placeHolderDrawable) {
        }
    };
    return target;
}

What's the best way to dedupe a table?

Deduping is rarely simple. That's because the records to be dedupped often have slightly different values is some of the fields. Therefore choose which record to keep can be problematic. Further, dups are often people records and it is hard to identify if the two John Smith's are two people or one person who is duplicated. So spend a lot (50% or more of the whole project) of your time defining what constitutes a dup and how to handle the differences and child records.

How do you know which is the correct value? Further dedupping requires that you handle all child records not orphaning any. What happens when you find that by changing the id on the child record you are suddenly violating one of the unique indexes or constraints - this will happen eventually and your process needs to handle it. If you have chosen foolishly to apply all your constraints only thorough the application, you may not even know the constraints are violated. When you have 10,000 records to dedup, you aren't going to go through the application to dedup one at a time. If the constraint isn't in the database, lots of luck in maintaining data integrity when you dedup.

A further complication is that dups don't always match exactly on the name or address. For instance a salesrep named Joan Martin may be a dup of a sales rep names Joan Martin-Jones especially if they have the same address and email. OR you could have John or Johnny in the name. Or the same street address except one record abbreveiated ST. and one spelled out Street. In SQL server you can use SSIS and fuzzy grouping to also identify near matches. These are often the most common dups as the fact that weren't exact matches is why they got put in as dups in the first place.

For some types of dedupping, you may need a user interface, so that the person doing the dedupping can choose which of two values to use for a particular field. This is especially true if the person who is being dedupped is in two or more roles. It could be that the data for a particular role is usually better than the data for another role. Or it could be that only the users will know for sure which is the correct value or they may need to contact people to find out if they are genuinely dups or simply two people with the same name.

Cannot GET / Nodejs Error

If you are getting this error, it could be because you don't have a route defined for your get.

For example:

const express = require('express');

const app = express();

app.get('/people', function (req, res) {
    res.send('hello');
})

app.listen(3000);


http://http://localhost:3000/people --> this works
http://http://localhost:3000 --> this will output Cannot GET / message.

nginx: connect() failed (111: Connection refused) while connecting to upstream

I don't think that solution would work anyways because you will see some error message in your error log file.

The solution was a lot easier than what I thought.

simply, open the following path to your php5-fpm

sudo nano /etc/php5/fpm/pool.d/www.conf

or if you're the admin 'root'

nano /etc/php5/fpm/pool.d/www.conf

Then find this line and uncomment it:

listen.allowed_clients = 127.0.0.1

This solution will make you be able to use listen = 127.0.0.1:9000 in your vhost blocks

like this: fastcgi_pass 127.0.0.1:9000;

after you make the modifications, all you need is to restart or reload both Nginx and Php5-fpm

Php5-fpm

sudo service php5-fpm restart

or

sudo service php5-fpm reload

Nginx

sudo service nginx restart

or

sudo service nginx reload

From the comments:

Also comment

;listen = /var/run/php5-fpm.sock 

and add

listen = 9000

Taking screenshot on Emulator from Android Studio

Starting with Android Studio 2.0 you can do it with the new emulator:

New Android Emulator from Android Studio 2.0

Just click 3 "Take Screenshot". Standard location is the desktop.

Or

  1. Select "More"
  2. Under "Settings", specify the location for your screenshot
  3. Take your screenshot

UPDATE 22/07/2020

If you keep the emulator in Android Studio as possible since Android Studio 4.1 click here to save the screenshot in your standard location:

enter image description here

MVC web api: No 'Access-Control-Allow-Origin' header is present on the requested resource

I know I'm coming to this very late. However, for anyone who's searching, I thought I'd publish what FINALLY worked for me. I'm not claiming it's the best solution - only that it worked.

Our WebApi service uses the config.EnableCors(corsAttribute) method. However, even with that, it would still fail on the pre-flight requests. @Mihai-Andrei Dinculescu's answer provided the clue for me. First of all, I added his Application_BeginRequest() code to flush the options requests. That STILL didn't work for me. The issue is that WebAPI still wasn't adding any of the expected headers to the OPTIONS request. Flushing it alone didn't work - but it gave me an idea. I added the custom headers that would otherwise be added via the web.config to the response for the OPTIONS request. Here's my code:

protected void Application_BeginRequest()
{
  if (Request.Headers.AllKeys.Contains("Origin") && Request.HttpMethod == "OPTIONS")
  {
    Response.Headers.Add("Access-Control-Allow-Origin", "https://localhost:44343");
    Response.Headers.Add("Access-Control-Allow-Headers",
      "Content-Type, Access-Control-Allow-Headers, Authorization, X-Requested-With");
    Response.Headers.Add("Access-Control-Allow-Methods", "GET, POST, PUT, DELETE, OPTIONS");
    Response.Headers.Add("Access-Control-Allow-Credentials", "true");
    Response.Flush();
  }
}

Obviously, this only applies to the OPTIONS requests. All other verbs are handled by the CORS configuration. If there's a better approach to this, I'm all ears. It feels like a cheat to me and I would prefer if the headers were added automatically, but this is what finally worked and allowed me to move on.

Simple JavaScript Checkbox Validation

For now no jquery or php needed. Use just "required" HTML5 input attrbute like here

 <form>
        <p>
            <input class="form-control" type="text" name="email" />
            <input type="submit" value="ok" class="btn btn-success" name="submit" />
            <input type="hidden" name="action" value="0" />
        </p>
        <p><input type="checkbox" required name="terms">I have read and accept <a href="#">SOMETHING Terms and Conditions</a></p>
 </form>

This will validate and prevent any submit before checkbox is opt in. Language independent solution because its generated by users web browser.

Trouble using ROW_NUMBER() OVER (PARTITION BY ...)

A bit involved. Easiest would be to refer to this SQL Fiddle I created for you that produces the exact result. There are ways you can improve it for performance or other considerations, but this should hopefully at least be clearer than some alternatives.

The gist is, you get a canonical ranking of your data first, then use that to segment the data into groups, then find an end date for each group, then eliminate any intermediate rows. ROW_NUMBER() and CROSS APPLY help a lot in doing it readably.


EDIT 2019:

The SQL Fiddle does in fact seem to be broken, for some reason, but it appears to be a problem on the SQL Fiddle site. Here's a complete version, tested just now on SQL Server 2016:

CREATE TABLE Source
(
  EmployeeID int,
  DateStarted date,
  DepartmentID int
)

INSERT INTO Source
VALUES
(10001,'2013-01-01',001),
(10001,'2013-09-09',001),
(10001,'2013-12-01',002),
(10001,'2014-05-01',002),
(10001,'2014-10-01',001),
(10001,'2014-12-01',001)


SELECT *, 
  ROW_NUMBER() OVER (PARTITION BY EmployeeID ORDER BY DateStarted) AS EntryRank,
  newid() as GroupKey,
  CAST(NULL AS date) AS EndDate
INTO #RankedData
FROM Source
;

UPDATE #RankedData
SET GroupKey = beginDate.GroupKey
FROM #RankedData sup
  CROSS APPLY 
  (
    SELECT TOP 1 GroupKey
    FROM #RankedData sub 
    WHERE sub.EmployeeID = sup.EmployeeID AND
      sub.DepartmentID = sup.DepartmentID AND
      NOT EXISTS 
        (
          SELECT * 
          FROM #RankedData bot 
          WHERE bot.EmployeeID = sup.EmployeeID AND
            bot.EntryRank BETWEEN sub.EntryRank AND sup.EntryRank AND
            bot.DepartmentID <> sup.DepartmentID
        )
      ORDER BY DateStarted ASC
    ) beginDate (GroupKey);

UPDATE #RankedData
SET EndDate = nextGroup.DateStarted
FROM #RankedData sup
  CROSS APPLY 
  (
    SELECT TOP 1 DateStarted
    FROM #RankedData sub
    WHERE sub.EmployeeID = sup.EmployeeID AND
      sub.DepartmentID <> sup.DepartmentID AND
      sub.EntryRank > sup.EntryRank
    ORDER BY EntryRank ASC
  ) nextGroup (DateStarted);

SELECT * FROM 
(
SELECT *, ROW_NUMBER() OVER (PARTITION BY GroupKey ORDER BY EntryRank ASC) AS GroupRank FROM #RankedData
) FinalRanking
WHERE GroupRank = 1
ORDER BY EntryRank;

DROP TABLE #RankedData
DROP TABLE Source

How to generate an entity-relationship (ER) diagram using Oracle SQL Developer

For a class diagram using Oracle database, use the following steps:

File ? Data Modeler ? Import ? Data Dictionary ? select DB connection ? Next ? select database->select tabels -> Finish

What is the difference between a mutable and immutable string in C#?

The data value may not be changed. Note: The variable value may be changed, but the original immutable data value was discarded and a new data value was created in memory.

Prevent flex items from stretching

You don't want to stretch the span in height?
You have the possiblity to affect one or more flex-items to don't stretch the full height of the container.

To affect all flex-items of the container, choose this:
You have to set align-items: flex-start; to div and all flex-items of this container get the height of their content.

_x000D_
_x000D_
div {_x000D_
  align-items: flex-start;_x000D_
  background: tan;_x000D_
  display: flex;_x000D_
  height: 200px;_x000D_
}_x000D_
span {_x000D_
  background: red;_x000D_
}
_x000D_
<div>_x000D_
  <span>This is some text.</span>_x000D_
</div>
_x000D_
_x000D_
_x000D_

To affect only a single flex-item, choose this:
If you want to unstretch a single flex-item on the container, you have to set align-self: flex-start; to this flex-item. All other flex-items of the container aren't affected.

_x000D_
_x000D_
div {_x000D_
  display: flex;_x000D_
  height: 200px;_x000D_
  background: tan;_x000D_
}_x000D_
span.only {_x000D_
  background: red;_x000D_
  align-self:flex-start;_x000D_
}_x000D_
span {_x000D_
    background:green;_x000D_
}
_x000D_
<div>_x000D_
  <span class="only">This is some text.</span>_x000D_
  <span>This is more text.</span>_x000D_
</div>
_x000D_
_x000D_
_x000D_

Why is this happening to the span?
The default value of the property align-items is stretch. This is the reason why the span fill the height of the div.

Difference between baseline and flex-start?
If you have some text on the flex-items, with different font-sizes, you can use the baseline of the first line to place the flex-item vertically. A flex-item with a smaller font-size have some space between the container and itself at top. With flex-start the flex-item will be set to the top of the container (without space).

_x000D_
_x000D_
div {_x000D_
  align-items: baseline;_x000D_
  background: tan;_x000D_
  display: flex;_x000D_
  height: 200px;_x000D_
}_x000D_
span {_x000D_
  background: red;_x000D_
}_x000D_
span.fontsize {_x000D_
  font-size:2em;_x000D_
}
_x000D_
<div>_x000D_
  <span class="fontsize">This is some text.</span>_x000D_
  <span>This is more text.</span>_x000D_
</div>
_x000D_
_x000D_
_x000D_

You can find more information about the difference between baseline and flex-start here:
What's the difference between flex-start and baseline?

How can I store and retrieve images from a MySQL database using PHP?

My opinion is, Instead of storing images directly to the database, It is recommended to store the image location in the database. As we compare both options, Storing images in the database is safe for security purpose. Disadvantage are

  1. If database is corrupted, no way to retrieve.

  2. Retrieving image files from db is slow when compare to other option.

On the other hand, storing image file location in db will have following advantages.

  1. It is easy to retrieve.

  2. If more than one images are stored, we can easily retrieve image information.

How do I force Maven to use my local repository rather than going out to remote repos to retrieve artifacts?

To truly force maven to only use your local repo, you can run with mvn <goals> -o. The -o tells maven to let you work "offline", and it will stay off the network.

What is apache's maximum url length?

The default limit for the length of the request line is 8192 bytes = 8* 1024. It you want to change the limit, you have to add or update in your tomcat server.xml the attribut maxHttpHeaderSize.

as:

<Connector port="8080" maxHttpHeaderSize="65536" protocol="HTTP/1.1" ... />

In this example I set the limite to 65536 bytes= 64*1024.

Hope this will help.

Convert any object to a byte[]

How about something simple like this?

return ((object[])value).Cast<byte>().ToArray(); 

Send mail via CMD console

A couple more command-line mailer programs:

Both support SSL too.

What's the difference between .NET Core, .NET Framework, and Xamarin?

.NET 5 will be a unified version of all .NET variants coming in November 2020, so there will be no need to choose between variants anymore. enter image description here

How to destroy a JavaScript object?

While the existing answers have given solutions to solve the issue and the second half of the question, they do not provide an answer to the self discovery aspect of the first half of the question that is in bold:

"How can I see which variable causes memory overhead...?"

It may not have been as robust 3 years ago, but the Chrome Developer Tools "Profiles" section is now quite powerful and feature rich. The Chrome team has an insightful article on using it and thus also how garbage collection (GC) works in javascript, which is at the core of this question.

Since delete is basically the root of the currently accepted answer by Yochai Akoka, it's important to remember what delete does. It's irrelevant if not combined with the concepts of how GC works in the next two answers: if there's an existing reference to an object it's not cleaned up. The answers are more correct, but probably not as appreciated because they require more thought than just writing 'delete'. Yes, one possible solution may be to use delete, but it won't matter if there's another reference to the memory leak.

Another answer appropriately mentions circular references and the Chrome team documentation can provide much more clarity as well as the tools to verify the cause.

Since delete was mentioned here, it also may be useful to provide the resource Understanding Delete. Although it does not get into any of the actual solution which is really related to javascript's garbage collector.

awk without printing newline

one way

awk '/^\*\*/{gsub("*","");printf "\n"$0" ";next}{printf $0" "}' to-plot.xls

What is the difference between public, private, and protected?

Visibility Scopes with Abstract Examples :: Makes easy Understanding

This visibility of a property or method is defined by pre-fixing declaration of one of three keyword (Public, protected and private)

Public : If a property or method is defined as public, it means it can be both access and manipulated by anything that can refer to object.

  • Abstract eg. Think public visibility scope as "public picnic" that anybody can come to.

Protected : when a property or method visibility is set to protected members can only be access within the class itself and by inherited & inheriting classes. (Inherited:- a class can have all the properties and methods of another class).

  • Think as a protected visibility scope as "Company picnic" where company members and their family members are allowed not the public. It's the most common scope restriction.

Private : When a property or method visibility is set to private, only the class that has the private members can access those methods and properties(Internally within the class), despite of whatever class relation there maybe.

  • with picnic analogy think as a "company picnic where only the company members are allowed" in the picnic. not family neither general public are allowed.

A method to count occurrences in a list

Your outer loop is looping over all the words in the list. It's unnecessary and will cause you problems. Remove it and it should work properly.

Why shouldn't I use PyPy over CPython if PyPy is 6.3 times faster?

The second question is easier to answer: you basically can use PyPy as a drop-in replacement if all your code is pure Python. However, many widely used libraries (including some of the standard library) are written in C and compiled as Python extensions. Some of these can be made to work with PyPy, some can't. PyPy provides the same "forward-facing" tool as Python --- that is, it is Python --- but its innards are different, so tools that interface with those innards won't work.

As for the first question, I imagine it is sort of a Catch-22 with the first: PyPy has been evolving rapidly in an effort to improve speed and enhance interoperability with other code. This has made it more experimental than official.

I think it's possible that if PyPy gets into a stable state, it may start getting more widely used. I also think it would be great for Python to move away from its C underpinnings. But it won't happen for a while. PyPy hasn't yet reached the critical mass where it is almost useful enough on its own to do everything you'd want, which would motivate people to fill in the gaps.

C++ cast to derived class

Think like this:

class Animal { /* Some virtual members */ };
class Dog: public Animal {};
class Cat: public Animal {};


Dog     dog;
Cat     cat;
Animal& AnimalRef1 = dog;  // Notice no cast required. (Dogs and cats are animals).
Animal& AnimalRef2 = cat;
Animal* AnimalPtr1 = &dog;
Animal* AnimlaPtr2 = &cat;

Cat&    catRef1 = dynamic_cast<Cat&>(AnimalRef1);  // Throws an exception  AnimalRef1 is a dog
Cat*    catPtr1 = dynamic_cast<Cat*>(AnimalPtr1);  // Returns NULL         AnimalPtr1 is a dog
Cat&    catRef2 = dynamic_cast<Cat&>(AnimalRef2);  // Works
Cat*    catPtr2 = dynamic_cast<Cat*>(AnimalPtr2);  // Works

// This on the other hand makes no sense
// An animal object is not a cat. Therefore it can not be treated like a Cat.
Animal  a;
Cat&    catRef1 = dynamic_cast<Cat&>(a);    // Throws an exception  Its not a CAT
Cat*    catPtr1 = dynamic_cast<Cat*>(&a);   // Returns NULL         Its not a CAT.

Now looking back at your first statement:

Animal   animal = cat;    // This works. But it slices the cat part out and just
                          // assigns the animal part of the object.
Cat      bigCat = animal; // Makes no sense.
                          // An animal is not a cat!!!!!
Dog      bigDog = bigCat; // A cat is not a dog !!!!

You should very rarely ever need to use dynamic cast.
This is why we have virtual methods:

void makeNoise(Animal& animal)
{
     animal.DoNoiseMake();
}

Dog    dog;
Cat    cat;
Duck   duck;
Chicken chicken;

makeNoise(dog);
makeNoise(cat);
makeNoise(duck);
makeNoise(chicken);

The only reason I can think of is if you stored your object in a base class container:

std::vector<Animal*>  barnYard;
barnYard.push_back(&dog);
barnYard.push_back(&cat);
barnYard.push_back(&duck);
barnYard.push_back(&chicken);

Dog*  dog = dynamic_cast<Dog*>(barnYard[1]); // Note: NULL as this was the cat.

But if you need to cast particular objects back to Dogs then there is a fundamental problem in your design. You should be accessing properties via the virtual methods.

barnYard[1]->DoNoiseMake();

JQuery create new select option

If you need to make single element you can use this construction:

$('<option/>', {
    'class': this.dataID,
    'text': this.s_dataValue
}).appendTo('.subCategory');

But if you need to print many elements you can use this construction:

function printOptions(arr){
    jQuery.each(arr, function(){
        $('<option/>', {
            'value': this.dataID,
            'text': this.s_dataValue
        }).appendTo('.subCategory');
    });
}

How to iterate object in JavaScript?

var dictionary = {
        "data":[{"id":"0","name":"ABC"}, {"id":"1","name":"DEF"}],
        "images": [ {"id":"0","name":"PQR"},"id":"1","name":"xyz"}]
};


for (var key in dictionary) {
    var getKey = dictionary[key];
    getKey.forEach(function(item) {
        console.log(item.name + ' ' + item.id);
    });
}

Visual Studio 2017: Display method references

In previous posts I have read that this feature IS available on VS 2015 community if you FIRST install SQL Server express (free) and THEN install VS. I have tried it and it worked. I just had to reinstall Windows and am going thru the same procedure now and it did not work... so will try again :). I know it worked 6 months ago when I tried.

-Ed

How to reverse apply a stash?

According to the git-stash manpage, "A stash is represented as a commit whose tree records the state of the working directory, and its first parent is the commit at HEAD when the stash was created," and git stash show -p gives us "the changes recorded in the stash as a diff between the stashed state and its original parent.

To keep your other changes intact, use git stash show -p | patch --reverse as in the following:

$ git init
Initialized empty Git repository in /tmp/repo/.git/

$ echo Hello, world >messages

$ git add messages

$ git commit -am 'Initial commit'
[master (root-commit)]: created 1ff2478: "Initial commit"
 1 files changed, 1 insertions(+), 0 deletions(-)
 create mode 100644 messages

$ echo Hello again >>messages

$ git stash

$ git status
# On branch master
nothing to commit (working directory clean)

$ git stash apply
# On branch master
# Changed but not updated:
#   (use "git add <file>..." to update what will be committed)
#   (use "git checkout -- <file>..." to discard changes in working directory)
#
#       modified:   messages
#
no changes added to commit (use "git add" and/or "git commit -a")

$ echo Howdy all >>messages

$ git diff
diff --git a/messages b/messages
index a5c1966..eade523 100644
--- a/messages
+++ b/messages
@@ -1 +1,3 @@
 Hello, world
+Hello again
+Howdy all

$ git stash show -p | patch --reverse
patching file messages
Hunk #1 succeeded at 1 with fuzz 1.

$ git diff
diff --git a/messages b/messages
index a5c1966..364fc91 100644
--- a/messages
+++ b/messages
@@ -1 +1,2 @@
 Hello, world
+Howdy all

Edit:

A light improvement to this is to use git apply in place of patch:

git stash show -p | git apply --reverse

Alternatively, you can also use git apply -R as a shorthand to git apply --reverse.

I've been finding this really handy lately...

How do I calculate a trendline for a graph?

Thanks to all for your help - I was off this issue for a couple of days and just came back to it - was able to cobble this together - not the most elegant code, but it works for my purposes - thought I'd share if anyone else encounters this issue:

public class Statistics
{
    public Trendline CalculateLinearRegression(int[] values)
    {
        var yAxisValues = new List<int>();
        var xAxisValues = new List<int>();

        for (int i = 0; i < values.Length; i++)
        {
            yAxisValues.Add(values[i]);
            xAxisValues.Add(i + 1);
        }

        return new Trendline(yAxisValues, xAxisValues);
    }
}

public class Trendline
{
    private readonly IList<int> xAxisValues;
    private readonly IList<int> yAxisValues;
    private int count;
    private int xAxisValuesSum;
    private int xxSum;
    private int xySum;
    private int yAxisValuesSum;

    public Trendline(IList<int> yAxisValues, IList<int> xAxisValues)
    {
        this.yAxisValues = yAxisValues;
        this.xAxisValues = xAxisValues;

        this.Initialize();
    }

    public int Slope { get; private set; }
    public int Intercept { get; private set; }
    public int Start { get; private set; }
    public int End { get; private set; }

    private void Initialize()
    {
        this.count = this.yAxisValues.Count;
        this.yAxisValuesSum = this.yAxisValues.Sum();
        this.xAxisValuesSum = this.xAxisValues.Sum();
        this.xxSum = 0;
        this.xySum = 0;

        for (int i = 0; i < this.count; i++)
        {
            this.xySum += (this.xAxisValues[i]*this.yAxisValues[i]);
            this.xxSum += (this.xAxisValues[i]*this.xAxisValues[i]);
        }

        this.Slope = this.CalculateSlope();
        this.Intercept = this.CalculateIntercept();
        this.Start = this.CalculateStart();
        this.End = this.CalculateEnd();
    }

    private int CalculateSlope()
    {
        try
        {
            return ((this.count*this.xySum) - (this.xAxisValuesSum*this.yAxisValuesSum))/((this.count*this.xxSum) - (this.xAxisValuesSum*this.xAxisValuesSum));
        }
        catch (DivideByZeroException)
        {
            return 0;
        }
    }

    private int CalculateIntercept()
    {
        return (this.yAxisValuesSum - (this.Slope*this.xAxisValuesSum))/this.count;
    }

    private int CalculateStart()
    {
        return (this.Slope*this.xAxisValues.First()) + this.Intercept;
    }

    private int CalculateEnd()
    {
        return (this.Slope*this.xAxisValues.Last()) + this.Intercept;
    }
}

Color text in terminal applications in UNIX

This is a little C program that illustrates how you could use color codes:

#include <stdio.h>

#define KNRM  "\x1B[0m"
#define KRED  "\x1B[31m"
#define KGRN  "\x1B[32m"
#define KYEL  "\x1B[33m"
#define KBLU  "\x1B[34m"
#define KMAG  "\x1B[35m"
#define KCYN  "\x1B[36m"
#define KWHT  "\x1B[37m"

int main()
{
    printf("%sred\n", KRED);
    printf("%sgreen\n", KGRN);
    printf("%syellow\n", KYEL);
    printf("%sblue\n", KBLU);
    printf("%smagenta\n", KMAG);
    printf("%scyan\n", KCYN);
    printf("%swhite\n", KWHT);
    printf("%snormal\n", KNRM);

    return 0;
}

adb doesn't show nexus 5 device

Answer by Rick and MadX is the right way to do the steps (Thumbs Up for the answer)

In my case I am using Akcess USB Type C Data Sync Cable For Nexus 5x, 5P - White As Nexus 5x do not supply type C to usb cable I purchased it from some vendor.

Having the same issue. What I am doing stupidly is:- I am connecting the cable in wrong way. After I reconnect it from upside down its working for me.

I might think that some of the Cables do not support debuggable. But its in my case.

This(Image) is my case the Type C should be as USB side symbol. A stupid solution, but work for me enter image description here

Fast check for NaN in NumPy

If you're comfortable with it allows to create a fast short-circuit (stops as soon as a NaN is found) function:

import numba as nb
import math

@nb.njit
def anynan(array):
    array = array.ravel()
    for i in range(array.size):
        if math.isnan(array[i]):
            return True
    return False

If there is no NaN the function might actually be slower than np.min, I think that's because np.min uses multiprocessing for large arrays:

import numpy as np
array = np.random.random(2000000)

%timeit anynan(array)          # 100 loops, best of 3: 2.21 ms per loop
%timeit np.isnan(array.sum())  # 100 loops, best of 3: 4.45 ms per loop
%timeit np.isnan(array.min())  # 1000 loops, best of 3: 1.64 ms per loop

But in case there is a NaN in the array, especially if it's position is at low indices, then it's much faster:

array = np.random.random(2000000)
array[100] = np.nan

%timeit anynan(array)          # 1000000 loops, best of 3: 1.93 µs per loop
%timeit np.isnan(array.sum())  # 100 loops, best of 3: 4.57 ms per loop
%timeit np.isnan(array.min())  # 1000 loops, best of 3: 1.65 ms per loop

Similar results may be achieved with Cython or a C extension, these are a bit more complicated (or easily avaiable as bottleneck.anynan) but ultimatly do the same as my anynan function.

How can I right-align text in a DataGridView column?

you can edit all the columns at once by using this simple code via Foreach loop

        foreach (DataGridViewColumn item in datagridview1.Columns)
        {
            item.DefaultCellStyle.Alignment = DataGridViewContentAlignment.MiddleRight;
        }

Location of GlassFish Server Logs

Locate the installation path of GlassFish. Then move to domains/domain-dir/logs/ and you'll find there the log files. If you have created the domain with NetBeans, the domain-dir is most probably called domain1.

See this link for the official GlassFish documentation about logging.

Difference in System. exit(0) , System.exit(-1), System.exit(1 ) in Java

Here is the answer.

System.exit(0);// normal termination - Successful - zero
System.exit(-1);//Exit with some Error
System.exit(1);//one or any positive integer // exit with some Information message

Using GitLab token to clone without authentication

As of 8.12, cloning using HTTPS + runner token is not supported anymore, as mentioned here:

In 8.12 we improved build permissions. Being able to clone project using runners token it is no supported from now on (it was actually working by coincidence and was never a fully fledged feature, so we changed that in 8.12). You should use build token instead.

This is widely documented here - https://docs.gitlab.com/ce/user/project/new_ci_build_permissions_model.html.

3 column layout HTML/CSS

.container{
height:100px;
width:500px;
border:2px dotted #F00;
border-left:none;
border-right:none;
text-align:center;
}
.container div{
display: inline-block;
border-left: 2px dotted #ccc;
vertical-align: middle;
line-height: 100px;
} 
 .column-left{ float: left; width: 32%;  height:100px;}
.column-right{ float: right; width: 32%;  height:100px; border-right: 2px dotted #ccc;}
.column-center{ display: inline-block; width: 33%; height:100px;}

 <div class="container">
   <div class="column-left">Column left</div>
   <div class="column-center">Column center</div>
   <div class="column-right">Column right</div>
</div> 

See this link http://jsfiddle.net/bipin_kumar/XD8RW/2/

LINQ to SQL: Multiple joins ON multiple Columns. Is this possible?

You can also join if the number of columns are not same in both tables and can map static value to table column

from t1 in Table1 
join t2 in Table2 
on new {X = t1.Column1, Y = 0 } on new {X = t2.Column1, Y = t2.Column2 }
select new {t1, t2}

HTTP GET request in JavaScript?

<button type="button" onclick="loadXMLDoc()"> GET CONTENT</button>

 <script>
        function loadXMLDoc() {
            var xmlhttp = new XMLHttpRequest();
            var url = "<Enter URL>";``
            xmlhttp.onload = function () {
                if (xmlhttp.readyState == 4 && xmlhttp.status == "200") {
                    document.getElementById("demo").innerHTML = this.responseText;
                }
            }
            xmlhttp.open("GET", url, true);
            xmlhttp.send();
        }
    </script>

Negation in Python

The negation operator in Python is not. Therefore just replace your ! with not.

For your example, do this:

if not os.path.exists("/usr/share/sounds/blues") :
    proc = subprocess.Popen(["mkdir", "/usr/share/sounds/blues"])
    proc.wait()

For your specific example (as Neil said in the comments), you don't have to use the subprocess module, you can simply use os.mkdir() to get the result you need, with added exception handling goodness.

Example:

blues_sounds_path = "/usr/share/sounds/blues"
if not os.path.exists(blues_sounds_path):
    try:
        os.mkdir(blues_sounds_path)
    except OSError:
        # Handle the case where the directory could not be created.

How to calculate time difference in java?

import java.util.Date;
...
Date d1 = new Date();
...
...
Date d2 = new Date();
System.out.println(d2.getTime()-d1.getTime()); //gives the time difference in milliseconds. 
System.out.println((d2.getTime()-d1.getTime())/1000); //gives the time difference in seconds.

and, to show in a nicer format, you can use:

    DecimalFormat myDecimalFormatter = new DecimalFormat("###,###.###");
    System.out.println(myDecimalFormatter.format(((double)d2.getTime()-d1.getTime())/1000));

PHP Array to CSV

It worked for me.

 $f=fopen('php://memory','w');
 $header=array("asdf ","asdf","asd","Calasdflee","Start Time","End Time" );      
 fputcsv($f,$header);
 fputcsv($f,$header);
 fputcsv($f,$header); 
 fseek($f,0);
 header('content-type:text/csv'); 
 header('Content-Disposition: attachment; filename="' . $filename . '";');
 fpassthru($f);```

https with WCF error: "Could not find base address that matches scheme https"

It turned out that my problem was that I was using a load balancer to handle the SSL, which then sent it over http to the actual server, which then complained.

Description of a fix is here: http://blog.hackedbrain.com/2006/09/26/how-to-ssl-passthrough-with-wcf-or-transportwithmessagecredential-over-plain-http/

Edit: I fixed my problem, which was slightly different, after talking to microsoft support.

My silverlight app had its endpoint address in code going over https to the load balancer. The load balancer then changed the endpoint address to http and to point to the actual server that it was going to. So on each server's web config I added a listenUri for the endpoint that was http instead of https

<endpoint address="" listenUri="http://[LOAD_BALANCER_ADDRESS]" ... />

Create Django model or update if exists

For only a small amount of objects the update_or_create works well, but if you're doing over a large collection it won't scale well. update_or_create always first runs a SELECT and thereafter an UPDATE.

for the_bar in bars:
    updated_rows = SomeModel.objects.filter(bar=the_bar).update(foo=100)
        if not updated_rows:
            # if not exists, create new
            SomeModel.objects.create(bar=the_bar, foo=100)

This will at best only run the first update-query, and only if it matched zero rows run another INSERT-query. Which will greatly increase your performance if you expect most of the rows to actually be existing.

It all comes down to your use case though. If you are expecting mostly inserts then perhaps the bulk_create() command could be an option.

'foo' was not declared in this scope c++

In C++, your source files are usually parsed from top to bottom in a single pass, so any variable or function must be declared before they can be used. There are some exceptions to this, like when defining functions inline in a class definition, but that's not the case for your code.

Either move the definition of integrate above the one for getSkewNormal, or add a forward declaration above getSkewNormal:

double integrate (double start, double stop, int numSteps, Evaluatable evalObj);

The same applies for sum.

Does WGET timeout?

According to the man page of wget, there are a couple of options related to timeouts -- and there is a default read timeout of 900s -- so I say that, yes, it could timeout.


Here are the options in question :

-T seconds
--timeout=seconds

Set the network timeout to seconds seconds. This is equivalent to specifying --dns-timeout, --connect-timeout, and --read-timeout, all at the same time.


And for those three options :

--dns-timeout=seconds

Set the DNS lookup timeout to seconds seconds.
DNS lookups that don't complete within the specified time will fail.
By default, there is no timeout on DNS lookups, other than that implemented by system libraries.

--connect-timeout=seconds

Set the connect timeout to seconds seconds.
TCP connections that take longer to establish will be aborted.
By default, there is no connect timeout, other than that implemented by system libraries.

--read-timeout=seconds

Set the read (and write) timeout to seconds seconds.
The "time" of this timeout refers to idle time: if, at any point in the download, no data is received for more than the specified number of seconds, reading fails and the download is restarted.
This option does not directly affect the duration of the entire download.


I suppose using something like

wget -O - -q -t 1 --timeout=600 http://www.example.com/cron/run

should make sure there is no timeout before longer than the duration of your script.

(Yeah, that's probably the most brutal solution possible ^^ )

Better solution without exluding fields from Binding

You should not use your domain models in your views. ViewModels are the correct way to do it.

You need to map your domain model's necessary fields to viewmodel and then use this viewmodel in your controllers. This way you will have the necessery abstraction in your application.

If you never heard of viewmodels, take a look at this.

Draw a curve with css

You could use an asymmetrical border to make curves with CSS.

border-radius: 50%/100px 100px 0 0;

VIEW DEMO

_x000D_
_x000D_
.box {_x000D_
  width: 500px; _x000D_
  height: 100px;  _x000D_
  border: solid 5px #000;_x000D_
  border-color: #000 transparent transparent transparent;_x000D_
  border-radius: 50%/100px 100px 0 0;_x000D_
}
_x000D_
<div class="box"></div>
_x000D_
_x000D_
_x000D_

How is a non-breaking space represented in a JavaScript string?

The jQuery docs for text() says

Due to variations in the HTML parsers in different browsers, the text returned may vary in newlines and other white space.

I'd use $td.html() instead.

How can I send JSON response in symfony2 controller

If your data is already serialized:

a) send a JSON response

public function someAction()
{
    $response = new Response();
    $response->setContent(file_get_contents('path/to/file'));
    $response->headers->set('Content-Type', 'application/json');
    return $response;
}

b) send a JSONP response (with callback)

public function someAction()
{
    $response = new Response();
    $response->setContent('/**/FUNCTION_CALLBACK_NAME(' . file_get_contents('path/to/file') . ');');
    $response->headers->set('Content-Type', 'text/javascript');
    return $response;
}

If your data needs be serialized:

c) send a JSON response

public function someAction()
{
    $response = new JsonResponse();
    $response->setData([some array]);
    return $response;
}

d) send a JSONP response (with callback)

public function someAction()
{
    $response = new JsonResponse();
    $response->setData([some array]);
    $response->setCallback('FUNCTION_CALLBACK_NAME');
    return $response;
}

e) use groups in Symfony 3.x.x

Create groups inside your Entities

<?php

namespace Mindlahus;

use Symfony\Component\Serializer\Annotation\Groups;

/**
 * Some Super Class Name
 *
 * @ORM    able("table_name")
 * @ORM\Entity(repositoryClass="SomeSuperClassNameRepository")
 * @UniqueEntity(
 *  fields={"foo", "boo"},
 *  ignoreNull=false
 * )
 */
class SomeSuperClassName
{
    /**
     * @Groups({"group1", "group2"})
     */
    public $foo;
    /**
     * @Groups({"group1"})
     */
    public $date;

    /**
     * @Groups({"group3"})
     */
    public function getBar() // is* methods are also supported
    {
        return $this->bar;
    }

    // ...
}

Normalize your Doctrine Object inside the logic of your application

<?php

use Symfony\Component\HttpFoundation\Response;
use Symfony\Component\Serializer\Mapping\Factory\ClassMetadataFactory;
// For annotations
use Doctrine\Common\Annotations\AnnotationReader;
use Symfony\Component\Serializer\Mapping\Loader\AnnotationLoader;
use Symfony\Component\Serializer\Serializer;
use Symfony\Component\Serializer\Normalizer\ObjectNormalizer;
use Symfony\Component\Serializer\Encoder\JsonEncoder;

...

$repository = $this->getDoctrine()->getRepository('Mindlahus:SomeSuperClassName');
$SomeSuperObject = $repository->findOneById($id);

$classMetadataFactory = new ClassMetadataFactory(new AnnotationLoader(new AnnotationReader()));
$encoder = new JsonEncoder();
$normalizer = new ObjectNormalizer($classMetadataFactory);
$callback = function ($dateTime) {
    return $dateTime instanceof \DateTime
        ? $dateTime->format('m-d-Y')
        : '';
};
$normalizer->setCallbacks(array('date' => $callback));
$serializer = new Serializer(array($normalizer), array($encoder));
$data = $serializer->normalize($SomeSuperObject, null, array('groups' => array('group1')));

$response = new Response();
$response->setContent($serializer->serialize($data, 'json'));
$response->headers->set('Content-Type', 'application/json');
return $response;

How can I set a DateTimePicker control to a specific date?

This oughta do it.

DateTimePicker1.Value = DateTime.Now.AddDays(-1).Date;

How to create a MySQL hierarchical recursive query?

Simple query to list child's of first recursion:

select @pv:=id as id, name, parent_id
from products
join (select @pv:=19)tmp
where parent_id=@pv

Result:

id  name        parent_id
20  category2   19
21  category3   20
22  category4   21
26  category24  22

... with left join:

select
    @pv:=p1.id as id
  , p2.name as parent_name
  , p1.name name
  , p1.parent_id
from products p1
join (select @pv:=19)tmp
left join products p2 on p2.id=p1.parent_id -- optional join to get parent name
where p1.parent_id=@pv

The solution of @tincot to list all child's:

select  id,
        name,
        parent_id 
from    (select * from products
         order by parent_id, id) products_sorted,
        (select @pv := '19') initialisation
where   find_in_set(parent_id, @pv) > 0
and     @pv := concat(@pv, ',', id)

Test it online with Sql Fiddle and see all results.

http://sqlfiddle.com/#!9/a318e3/4/0

What is the right way to write my script 'src' url for a local development environment?

This is an old post but...

You can reference the working directory (the folder the .html file is located in) with ./, and the directory above that with ../

Example directory structure:

/html/public/
- index.html
- script2.js
- js/
   - script.js

To load script.js from inside index.html:

<script type="text/javascript" src="./js/script.js">

This goes to the current working directory (location of index.html) and then to the js folder, and then finds the script.

You could also specify ../ to go one directory above the working directory, to load things from there. But that is unusual.

Fastest method to replace all instances of a character in a string

Also you can try:

string.split('foo').join('bar');

How to use paths in tsconfig.json?

It looks like there has been an update to React that doesn't allow you to set the "paths" in the tsconfig.json anylonger.

Nicely React just outputs a warning:

The following changes are being made to your tsconfig.json file:
  - compilerOptions.paths must not be set (aliased imports are not supported)

then updates your tsconfig.json and removes the entire "paths" section for you. There is a way to get around this run

npm run eject

This will eject all of the create-react-scripts settings by adding config and scripts directories and build/config files into your project. This also allows a lot more control over how everything is built, named etc. by updating the {project}/config/* files.

Then update your tsconfig.json

{
    "compilerOptions": {
        "baseUrl": "./src",
        {…}
        "paths": {
            "assets/*": [ "assets/*" ],
            "styles/*": [ "styles/*" ]
        }
    },
}

Hibernate error - QuerySyntaxException: users is not mapped [from users]

with org.hibernate.hql.internal.ast.QuerySyntaxException: users is not mapped [from users], you are trying to select from the users table. But you are annotating your class with @Table( name = "Users" ). So either use users, or Users.

Best practices when running Node.js with port 80 (Ubuntu / Linode)

Drop root privileges after you bind to port 80 (or 443).

This allows port 80/443 to remain protected, while still preventing you from serving requests as root:

function drop_root() {
    process.setgid('nobody');
    process.setuid('nobody');
}

A full working example using the above function:

var process = require('process');
var http = require('http');
var server = http.createServer(function(req, res) {
    res.write("Success!");
    res.end();
});

server.listen(80, null, null, function() {
    console.log('User ID:',process.getuid()+', Group ID:',process.getgid());
    drop_root();
    console.log('User ID:',process.getuid()+', Group ID:',process.getgid());
});

See more details at this full reference.

Adding a new line/break tag in XML

Had same issue when I had to develop a fixed length field format.

Usually we do not use line separator for binary files but For some reason our customer wished to add a line break as separator between records. They set

< record_delimiter value="\n"/ >

but this didn't work as records got two additional characters:
< record1 > \n < record2 > \n.... and so on.

Did following change and it just worked.

< record_delimiter value="\n"/> => < record_delimiter value="&#xA;"/ >

After unmarshaling Java interprets as new line character.

Ruby on Rails generates model field:type - what are the options for field:type?

:primary_key, :string, :text, :integer, :float, :decimal, :datetime, :timestamp,
:time, :date, :binary, :boolean, :references

See the table definitions section.

Can I hide the HTML5 number input’s spin box?

Short answer:

_x000D_
_x000D_
input[type="number"]::-webkit-outer-spin-button,
input[type="number"]::-webkit-inner-spin-button {
    -webkit-appearance: none;
    margin: 0;
}
input[type="number"] {
    -moz-appearance: textfield;
}
_x000D_
<input type="number" />
_x000D_
_x000D_
_x000D_

Longer answer:

To add to existing answer...

Firefox:

In current versions of Firefox, the (user agent) default value of the -moz-appearance property on these elements is number-input. Changing that to the value textfield effectively removes the spinner.

input[type="number"] {
    -moz-appearance: textfield;
}

In some cases, you may want the spinner to be hidden initially, and then appear on hover/focus. (This is currently the default behavior in Chrome). If so, you can use the following:

_x000D_
_x000D_
input[type="number"] {
    -moz-appearance: textfield;
}
input[type="number"]:hover,
input[type="number"]:focus {
    -moz-appearance: number-input;
}
_x000D_
<input type="number"/>
_x000D_
_x000D_
_x000D_


Chrome:

In current versions of Chrome, the (user agent) default value of the -webkit-appearance property on these elements is already textfield. In order to remove the spinner, the -webkit-appearance property's value needs to be changed to none on the ::-webkit-outer-spin-button/::-webkit-inner-spin-button pseudo classes (it is -webkit-appearance: inner-spin-button by default).

_x000D_
_x000D_
input[type="number"]::-webkit-outer-spin-button,
input[type="number"]::-webkit-inner-spin-button {
    -webkit-appearance: none;
    margin: 0;
}
_x000D_
<input type="number" />
_x000D_
_x000D_
_x000D_

It's worth pointing out that margin: 0 is used to remove the margin in older versions of Chrome.

Currently, as of writing this, here is the default user agent styling on the 'inner-spin-button' pseudo class:

input::-webkit-inner-spin-button {
    -webkit-appearance: inner-spin-button;
    display: inline-block;
    cursor: default;
    flex: 0 0 auto;
    align-self: stretch;
    -webkit-user-select: none;
    opacity: 0;
    pointer-events: none;
    -webkit-user-modify: read-only;
}

Accept function as parameter in PHP

Simple example using a class :

class test {

    public function works($other_parameter, $function_as_parameter)
    {

        return $function_as_parameter($other_parameter) ;

    }

}

$obj = new test() ;

echo $obj->works('working well',function($other_parameter){


    return $other_parameter;


});

Where should my npm modules be installed on Mac OS X?

npm root -g

to check the npm_modules global location

How to plot an array in python?

if you give a 2D array to the plot function of matplotlib it will assume the columns to be lines:

If x and/or y is 2-dimensional, then the corresponding columns will be plotted.

In your case your shape is not accepted (100, 1, 1, 8000). As so you can using numpy squeeze to solve the problem quickly:

np.squeez doc: Remove single-dimensional entries from the shape of an array.

import numpy as np
import matplotlib.pyplot as plt

data = np.random.randint(3, 7, (10, 1, 1, 80))
newdata = np.squeeze(data) # Shape is now: (10, 80)
plt.plot(newdata) # plotting by columns
plt.show()

But notice that 100 sets of 80 000 points is a lot of data for matplotlib. I would recommend that you look for an alternative. The result of the code example (run in Jupyter) is:

Jupyter matplotlib plot

Get the position of a spinner in Android

if (position ==0) {
    if (rYes.isChecked()) {
        Toast.makeText(SportActivity.this, "yes ur answer is right", Toast.LENGTH_LONG).show();
    } else if (rNo.isChecked()) {
        Toast.makeText(SportActivity.this, "no.ur answer is wrong", Toast.LENGTH_LONG).show();
    }
}

This code is supposed to select both check boxes.
Is there a problem with it?

Effective method to hide email from spam bots

First I would make sure the email address only shows when you have javascript enabled. This way, there is no plain text that can be read without javascript.

Secondly, A way of implementing a safe feature is by staying away from the <button> tag. This tag needs a text insert between the tags, which makes it computer-readable. Instead try the <input type="button"> with a javascript handler for an onClick. Then use all of the techniques mentioned by otherse to implement a safe email notation.

One other option is to have a button with "Click to see emailaddress". Once clicked this changes into a coded email (the characters in HTML codes). On another click this redirects to the 'mailto:email' function

An uncoded version of the last idea, with selectable and non-selectable email addresses:

<html>
<body>
<script type="text/javascript">
      e1="@domain";
      e2="me";
      e3=".extension";
email_link="mailto:"+e2+e1+e3;
</script>
<input type="text" onClick="this.onClick=window.open(email_link);" value="Click for mail"/>
<input type="text" onClick="this.value=email;" value="Click for mail-address"/>
<input type="button" onClick="this.onClick=window.open(email_link);" value="Click for mail"/>
<input type="button" onClick="this.value=email;" value="Click for mail-address"/>
</body></html>

See if this is something you would want and combine it with others' ideas. You can never be too sure.

Get the date of next monday, tuesday, etc

For some reason, strtotime('next friday') display the Friday date of the current week. Try this instead:

//Current date 2020-02-03
$fridayNextWeek = date('Y-m-d', strtotime('friday next week'); //Outputs 2020-02-14

$nextFriday = date('Y-m-d', strtotime('next friday'); //Outputs 2020-02-07

android adb turn on wifi via adb

In my ROM, it's stored in the "global" database, rather than "secure". So D__'s answer is correct, but the sql line needs to connect to a different database:

sqlite3 /data/data/com.android.providers.settings/databases/settings.db
update global set value=1 where name='wifi_on';

How to extend available properties of User.Identity

Whenever you want to extend the properties of User.Identity with any additional properties like the question above, add these properties to the ApplicationUser class first like so:

public class ApplicationUser : IdentityUser
{
    public async Task<ClaimsIdentity> GenerateUserIdentityAsync(UserManager<ApplicationUser> manager)
    {
        // Note the authenticationType must match the one defined in CookieAuthenticationOptions.AuthenticationType
        var userIdentity = await manager.CreateIdentityAsync(this, DefaultAuthenticationTypes.ApplicationCookie);
        // Add custom user claims here
        return userIdentity;
    }

    // Your Extended Properties
    public long? OrganizationId { get; set; }
}

Then what you need is to create an extension method like so (I create mine in an new Extensions folder):

namespace App.Extensions
{
    public static class IdentityExtensions
    {
        public static string GetOrganizationId(this IIdentity identity)
        {
            var claim = ((ClaimsIdentity)identity).FindFirst("OrganizationId");
            // Test for null to avoid issues during local testing
            return (claim != null) ? claim.Value : string.Empty;
        }
    }
}

When you create the Identity in the ApplicationUser class, just add the Claim -> OrganizationId like so:

    public async Task<ClaimsIdentity> GenerateUserIdentityAsync(UserManager<ApplicationUser> manager)
    {
        // Note the authenticationType must match the one defined in CookieAuthenticationOptions.AuthenticationType
        var userIdentity = await manager.CreateIdentityAsync(this, DefaultAuthenticationTypes.ApplicationCookie);
        // Add custom user claims here => this.OrganizationId is a value stored in database against the user
        userIdentity.AddClaim(new Claim("OrganizationId", this.OrganizationId.ToString()));

        return userIdentity;
    }

Once you added the claim and have your extension method in place, to make it available as a property on your User.Identity, add a using statement on the page/file you want to access it:

in my case: using App.Extensions; within a Controller and @using. App.Extensions withing a .cshtml View file.

EDIT:

What you can also do to avoid adding a using statement in every View is to go to the Views folder, and locate the Web.config file in there. Now look for the <namespaces> tag and add your extension namespace there like so:

<add namespace="App.Extensions" />

Save your file and you're done. Now every View will know of your extensions.

You can access the Extension Method:

var orgId = User.Identity.GetOrganizationId();

Merging cells in Excel using Apache POI

syntax is:

sheet.addMergedRegion(new CellRangeAddress(start-col,end-col,start-cell,end-cell));

Example:

sheet.addMergedRegion(new CellRangeAddress(4, 4, 0, 5));

Here the cell 0 to cell 5 will be merged of the 4th row.

How to fix Git error: object file is empty?

Because I have to reboot my VM regularly, so somehow this problem happens to me very often. After few times of it, I realized I cannot repeat the process described by @Nathan-Vanhoudnos every time this happens, though it always works. Then I figured out the following faster solution.

Step 1

Move your entire repo to another folder.

mv current_repo temp_repo

Step 2

Clone the repo from origin again.

git clone source_to_current_repo.git

Step 3

Remove Everything under the new repo except the .git folder.

Step 4

Move Everything from the temp_repo to the new repo except the .git folder.

Step 5

Remove the temp_repo, and we are done.

After few times, I'm sure you can do this procedures very quickly.

Converting String To Float in C#

The precision of float is 7 digits. If you want to keep the whole lot, you need to use the double type that keeps 15-16 digits. Regarding formatting, look at a post about formatting doubles. And you need to worry about decimal separators in C#.

C# declare empty string array

Try this

string[] arr = new string[] {};

How do I get the SelectedItem or SelectedIndex of ListView in vb.net?

ListView.FocusedItem.Index 

or you can use foreach loop like this

int index= -1;
foreach (ListViewItem itm in listView1.SelectedItems)
{
    if (itm.Selected)
    {
        index= itm.Index;
    }
}

A warning - comparison between signed and unsigned integer expressions

The important difference between signed and unsigned ints is the interpretation of the last bit. The last bit in signed types represent the sign of the number, meaning: e.g:

0001 is 1 signed and unsigned 1001 is -1 signed and 9 unsigned

(I avoided the whole complement issue for clarity of explanation! This is not exactly how ints are represented in memory!)

You can imagine that it makes a difference to know if you compare with -1 or with +9. In many cases, programmers are just too lazy to declare counting ints as unsigned (bloating the for loop head f.i.) It is usually not an issue because with ints you have to count to 2^31 until your sign bit bites you. That's why it is only a warning. Because we are too lazy to write 'unsigned' instead of 'int'.

How to Install Sublime Text 3 using Homebrew

$ brew tap caskroom/cask
$ brew install brew-cask
$ brew tap caskroom/versions
$ brew cask install sublime-text

comparing elements of the same array in java

Try this or purpose will solve with lesser no of steps

for (int i = 0; i < a.length; i++) 
{
    for (int k = i+1; k < a.length; k++) 
    {
        if (a[i] != a[k]) 
         {
            System.out.println(a[i]+"not the same with"+a[k]+"\n");
        }
    }
}

PostgreSQL: How to change PostgreSQL user password?

For my case on Ubuntu 14.04 installed with postgres 10.3. I need to follow the following steps

  • su - postgres to switch user to postgres
  • psql to enter postgres shell
  • \password then enter your password
  • \q to quit the shell session
  • Then you switch back to root by executing exit and configure your pg_hba.conf (mine is at /etc/postgresql/10/main/pg_hba.conf) by making sure you have the following line

    local all postgres md5

  • Restart your postgres service by service postgresql restart
  • Now switch to postgres user and enter postgres shell again. It will prompt you with password.

How to use matplotlib tight layout with Figure?

Just call fig.tight_layout() as you normally would. (pyplot is just a convenience wrapper. In most cases, you only use it to quickly generate figure and axes objects and then call their methods directly.)

There shouldn't be a difference between the QtAgg backend and the default backend (or if there is, it's a bug).

E.g.

import matplotlib.pyplot as plt

#-- In your case, you'd do something more like:
# from matplotlib.figure import Figure
# fig = Figure()
#-- ...but we want to use it interactive for a quick example, so 
#--    we'll do it this way
fig, axes = plt.subplots(nrows=4, ncols=4)

for i, ax in enumerate(axes.flat, start=1):
    ax.set_title('Test Axes {}'.format(i))
    ax.set_xlabel('X axis')
    ax.set_ylabel('Y axis')

plt.show()

Before Tight Layout

enter image description here

After Tight Layout

import matplotlib.pyplot as plt

fig, axes = plt.subplots(nrows=4, ncols=4)

for i, ax in enumerate(axes.flat, start=1):
    ax.set_title('Test Axes {}'.format(i))
    ax.set_xlabel('X axis')
    ax.set_ylabel('Y axis')

fig.tight_layout()

plt.show()

enter image description here

how to convert a string to a bool

Here's my attempt at the most forgiving string to bool conversion that is still useful, basically keying off only the first character.

public static class StringHelpers
{
    /// <summary>
    /// Convert string to boolean, in a forgiving way.
    /// </summary>
    /// <param name="stringVal">String that should either be "True", "False", "Yes", "No", "T", "F", "Y", "N", "1", "0"</param>
    /// <returns>If the trimmed string is any of the legal values that can be construed as "true", it returns true; False otherwise;</returns>
    public static bool ToBoolFuzzy(this string stringVal)
    {
        string normalizedString = (stringVal?.Trim() ?? "false").ToLowerInvariant();
        bool result = (normalizedString.StartsWith("y") 
            || normalizedString.StartsWith("t")
            || normalizedString.StartsWith("1"));
        return result;
    }
}

Set Background cell color in PHPExcel

This code should work for you:

 $PHPExcel->getActiveSheet()
        ->getStyle('A1')
        ->getFill()
        ->setFillType(PHPExcel_Style_Fill::FILL_SOLID)
        ->getStartColor()
        ->setRGB('FF0000')

But if you bother using this over and over again, I recommend using applyFromArray.

Xcode Error: "The app ID cannot be registered to your development team."

The bundle id (app ID) has a binding relationship with the apple id (apple id is the Apple development account, which also belongs to a certain development team). When the app is created, the bundle id (app ID) is already associated with you The development team is bound, so your app is being sent to other colleagues, he opens it in Xcode, and connects the real machine with a data cable to debug it will report the error as above;

To Solution

  1. Follow the prompts to change the bundle id

    enter image description here

  2. Because some functions, such as third-party login, are bound to the bundle id to apply for the app key, etc., the bundle id cannot be modified. Then please find the apple id account registered by the bundle id before. Here, I will change It’s ok to become the apple id account I registered in the company group

  3. If other colleagues in your group can run this app successfully on a real machine, it means that the "description file" corresponding to the apple id "certificate" used by him is correct.

How to check the input is an integer or not in Java?

Using Integer.parseIn(String), you can parse string value into integer. Also you need to catch exception in case if input string is not a proper number.

int x = 0;

try {       
    x = Integer.parseInt("100"); // Parse string into number
} catch (NumberFormatException e) {
    e.printStackTrace();
}

jQuery plugin returning "Cannot read property of undefined"

I had same problem with 'parallax' plugin. I changed jQuery librery version to *jquery-1.6.4* from *jquery-1.10.2*. And error cleared.

How do I filter ForeignKey choices in a Django ModelForm?

ForeignKey is represented by django.forms.ModelChoiceField, which is a ChoiceField whose choices are a model QuerySet. See the reference for ModelChoiceField.

So, provide a QuerySet to the field's queryset attribute. Depends on how your form is built. If you build an explicit form, you'll have fields named directly.

form.rate.queryset = Rate.objects.filter(company_id=the_company.id)

If you take the default ModelForm object, form.fields["rate"].queryset = ...

This is done explicitly in the view. No hacking around.

How to determine the installed webpack version

For those who are using yarn

yarn list webpack will do the trick

$ yarn list webpack
yarn list v0.27.5
+- [email protected]
Done in 1.24s.

How can I run an external command asynchronously from Python?

subprocess.Popen does exactly what you want.

from subprocess import Popen
p = Popen(['watch', 'ls']) # something long running
# ... do other stuff while subprocess is running
p.terminate()

(Edit to complete the answer from comments)

The Popen instance can do various other things like you can poll() it to see if it is still running, and you can communicate() with it to send it data on stdin, and wait for it to terminate.

How to calculate combination and permutation in R?

The Combinations package is not part of the standard CRAN set of packages, but is rather part of a different repository, omegahat. To install it you need to use

install.packages("Combinations", repos = "http://www.omegahat.org/R")

See the documentation at http://www.omegahat.org/Combinations/

How to open a PDF file in an <iframe>?

Do it like this: Remember to close iframe tag.

<iframe src="http://samplepdf.com/sample.pdf" width="800" height="600"></iframe>

python: iterate a specific range in a list

You want to use slicing.

for item in listOfStuff[1:3]:
    print item

Format datetime in asp.net mvc 4

Ahhhh, now it is clear. You seem to have problems binding back the value. Not with displaying it on the view. Indeed, that's the fault of the default model binder. You could write and use a custom one that will take into consideration the [DisplayFormat] attribute on your model. I have illustrated such a custom model binder here: https://stackoverflow.com/a/7836093/29407


Apparently some problems still persist. Here's my full setup working perfectly fine on both ASP.NET MVC 3 & 4 RC.

Model:

public class MyViewModel
{
    [DisplayName("date of birth")]
    [DataType(DataType.Date)]
    [DisplayFormat(DataFormatString = "{0:dd/MM/yyyy}", ApplyFormatInEditMode = true)]
    public DateTime? Birth { get; set; }
}

Controller:

public class HomeController : Controller
{
    public ActionResult Index()
    {
        return View(new MyViewModel
        {
            Birth = DateTime.Now
        });
    }

    [HttpPost]
    public ActionResult Index(MyViewModel model)
    {
        return View(model);
    }
}

View:

@model MyViewModel

@using (Html.BeginForm())
{
    @Html.LabelFor(x => x.Birth)
    @Html.EditorFor(x => x.Birth)
    @Html.ValidationMessageFor(x => x.Birth)
    <button type="submit">OK</button>
}

Registration of the custom model binder in Application_Start:

ModelBinders.Binders.Add(typeof(DateTime?), new MyDateTimeModelBinder());

And the custom model binder itself:

public class MyDateTimeModelBinder : DefaultModelBinder
{
    public override object BindModel(ControllerContext controllerContext, ModelBindingContext bindingContext)
    {
        var displayFormat = bindingContext.ModelMetadata.DisplayFormatString;
        var value = bindingContext.ValueProvider.GetValue(bindingContext.ModelName);

        if (!string.IsNullOrEmpty(displayFormat) && value != null)
        {
            DateTime date;
            displayFormat = displayFormat.Replace("{0:", string.Empty).Replace("}", string.Empty);
            // use the format specified in the DisplayFormat attribute to parse the date
            if (DateTime.TryParseExact(value.AttemptedValue, displayFormat, CultureInfo.InvariantCulture, DateTimeStyles.None, out date))
            {
                return date;
            }
            else
            {
                bindingContext.ModelState.AddModelError(
                    bindingContext.ModelName,
                    string.Format("{0} is an invalid date format", value.AttemptedValue)
                );
            }
        }

        return base.BindModel(controllerContext, bindingContext);
    }
}

Now, no matter what culture you have setup in your web.config (<globalization> element) or the current thread culture, the custom model binder will use the DisplayFormat attribute's date format when parsing nullable dates.

codeigniter, result() vs. result_array()

result_array() returns Associative Array type data. Returning pure array is slightly faster than returning an array of objects. result() is recursive in that it returns an std class object where as result_array() just returns a pure array, so result_array() would be choice regarding performance.

X11/Xlib.h not found in Ubuntu

Andrew White's answer is sufficient to get you moving. Here's a step-by-step for beginners.

A simple get started:

Create test.cpp: (This will be built and run to verify you got things set up right.)

#include <X11/Xlib.h>
#include <unistd.h>


main()
{
  // Open a display.
  Display *d = XOpenDisplay(0);

  if ( d )
    {
      // Create the window
      Window w = XCreateWindow(d, DefaultRootWindow(d), 0, 0, 200,
                   100, 0, CopyFromParent, CopyFromParent,
                   CopyFromParent, 0, 0);

      // Show the window
      XMapWindow(d, w);
      XFlush(d);

      // Sleep long enough to see the window.
      sleep(10);
    }
  return 0;
}

(Source: LinuxGazette)

Try: g++ test.cpp -lX11 If it builds to a.out, try running it. If you see a simple window drawn, you have the necessary libraries, and some other root problem is afoot.

If your response is:

    test.cpp:1:22: fatal error: X11/Xlib.h: No such file or directory
    compilation terminated.

you need to install X11 development libraries. sudo apt-get install libx11-dev

Retry g++ test.cpp -lX11

If it works, you're golden.

Tested using a fresh install of libX11-dev_2%3a1.5.0-1_i386.deb